Самые сложные загадки с ответами для 11 лет: Математика и логика для детей — занимательная математика для детей 7-13 лет

Содержание

Загадки с подвохом и на логику

Во Франции и Германии это означает ноль, в Японии — деньги, в Тунисе — угроза смерти, в Сирии — разрыв отношений. А в США?
Ответ: ОК (знак, который можно показать пальцами)
1634

В каком случае 10 взрослых и 10 детей, забравшись под один зонтик, не намокнут?
Ответ: Когда нет дождя
2364

Сидит человек. А вы не можете сесть на его место, даже если он встанет и уйдёт. Где он сидит?
Ответ: На ваших коленях
3586

Когда человек бывает в комнате без головы?
Ответ: Когда он высовывает голову через окно на улицу
5297

Что чем больше ешь, тем больше остаётся?
Ответ: Орех, скорлупа
5271

Что можно видеть с закрытыми глазами?
Ответ: Сон
5660

Не было и не будет, а названье было и будет.
Ответ: Птичье молоко
5186

У трёх шоферов был брат Андрей, а у Андрея братьев не было. Могло ли это быть?
Ответ: Да, если шофёры были женщинами
3767

Что может в одно и то же время

Висеть и стоять, стоять и ходить,
Ходить и лежать, лежать и врать?
Ответ: Часы
3842

Какая одежда состоит из ста частей без единого шва?
Ответ: Перо, пух
4631

Кто без чего не обойдётся?
Ответ: Человек без имени
5144

На какой вопрос никто никогда не ответит «да»?
Ответ: Спящий на вопрос: «Вы спите?»
Безруков Александр, Рассказово
1630

По какому пути никто не ходил и не ездил?
Ответ: По млечному
3680

Что за трава, которую даже слепые знают?
Ответ: Крапива
4309

Какой месяц короче всех?
Ответ: Май (три буквы)
4068

Корова — 2, кошка — 3, овца — 2, кукушка — 4, петух — 8, собака — 3, осёл — ?
Ответ: 2 (иа)
5391

На что больше всего походит половина апельсина?
Ответ: На другую половину апельсина
3754

В каком месяце люди едят меньше всего?
Ответ: В феврале
4280

Если бросить в Чёрное море красный камень, то какой он будет?

Ответ: Мокрый
3261

Какой огонь не жжёт?
Ответ: Нарисованный
4625

Чем больше из неё берёшь, тем больше она становится.
Ответ: Яма
3130

Шесть ног, а бежит не быстрее, чем на четырёх.
Ответ: Всадник на коне
3431

Какая еда, хоть вари её в пяти пудах соли, никогда не будет солёной?
Ответ: Яйцо
2736

Что вчера было «завтра», а завтра будет «вчера»?
Ответ: Сегодня
3508

Можно ли в решете воды принести?
Ответ: Можно, когда она замёрзнет
3825

Сколько горошин может войти в обыкновенный стакан?
Ответ: Сами они войти не могут
2998

Может ли страус назвать себя птицей?
Ответ: Нет, он не умеет говорить
3462

Когда руки бывают тремя местоимениями?
Ответ: Когда они вы-мы-ты
3269

Какое слово всегда звучит неверно?
Ответ: Слово «неверно»
3527

На какое дерево садится ворона во время дождя?
Ответ: На мокрое
2682

Сын моего отца, а мне не брат. Кто это?
Ответ: Я сам
3752

Когда в пустом кармане есть что-нибудь?
Ответ: Когда в кармане дырка
3199

Из какой посуды нельзя ничего поесть?
Ответ: Из пустой
2941

Шёл дождь. Ехал автобус. В автобусе все спали и только водитель не спал. Какой номер автобуса и как звали водителя?
Ответ: номер автобуса мокрый, водителя звали Толька ~ “только водитель”
3597

А и б сидели на трубе, а — упало, б — пропало. Что осталось на трубе?
Ответ: Осталась и
2879

Какие часы показывают правильное время только два раза в сутки?
Ответ: Сломанные механические
2607

В названии какого города имя одного мальчика и имя ста девочек?
Ответ: Севастополь
2727

Когда мужчину называют женским именем?

Ответ: Когда он спит. Соня
3124

У сороки две, у человека одна, у медведя ни одной.
Ответ: Буква «о»
2838

Что не может поместиться в самую большую кастрюлю?
Ответ: Крышка от этой кастрюли
Приставка Мария
1249

Сколько яиц можно съесть натощак?
Ответ: Одно, остальные уже не натощак
3174

Чем заканчивается день и ночь?
Ответ: Буквой «ь»
2951

Страницы: 1 2 … 9 10 11

Если загадок с подвохом вам оказалось мало, посмотрите загадки схожей тематики: прикольные и смешные загадки.
На тему загадок с подвохом для вас может быть интересной наша статья «Как нарисовать 1000 человечков за одну минуту?»



Загадки для детей 8 лет: 100 увлекательных заданий

Загадки для детей 8 лет с ответами увлекут вашего ребенка, интересные загадки развивают природное чувство любопытства у детей. Когда найден правильный ответ, ребенок радуется этой маленькой победе.

Дети в возрасте 8 лет очень любят различные загадки.

Содержание статьи:

Загадка для детей 8 лет с ответами

Ниже представлены загадки для детей 8 лет с ответами.

Загадка №1

На носу сидим,
На мир глядим,
За уши держимся.

ВНИМАНИЕ! ЗАПИСЬ НА КУРСЫ! ИДЕТ НАБОР!

Подробнее на странице: https://academy.multi-mama.ru/product/multi-predlozhenie/

(Ответ: Очки)

***

Загадка №2

Говорит она беззвучно,
А понятно и нескучно.
Ты беседуй чаще с ней —
Станешь вчетверо умней.

(Ответ: Книга)

***

Загадка №3

Без него домой придёшь —
Точно в дом не попадёшь.

(Ответ: Ключ)

***

Загадка №4

Есть в комнате портрет,
Во всём на вас похожий.
Засмейтесь — и в ответ
Он засмеется тоже.

(Ответ: Зеркало)

***

Загадка №5

Себя он раскрывает,
Тебя он закрывает.

(Ответ: Зонт)

***

Загадка №6

Из горячего колодца
Через нос водица льётся.

(Ответ: Чайник)

***

Загадка №7

Живёт он на кухне,
Проворный, худой,
Он пол подметает
Своей бородой.

(Ответ: Веник)

***

Загадка №8

Вот гора, а у горы
Две глубокие норы.
В этих норах воздух бродит,
То заходит, то выходит.

(Ответ: Нос)

***

Загадка №9

Дали братьям тёплый дом,
Чтобы жили впятером.
Брат большой не согласился
И отдельно поселился.

(Ответ: Варежки)

***

Загадка №10

Два братца не могут расстаться:
Утром — в дороге,
Ночью — на пороге.

(Ответ: Ботинки)

***

Загадка №11

Зорко смотрит постовой
За широкой мостовой.
Как посмотрит глазом красным —
Остановятся все сразу.

(Ответ: Светофор)

***

Смотрите также:

Загадки для детей: 125 загадок с ответами для детей 3-4 лет
Загадки для детей: 70 лучших загадок для детей 4-5 лет
Стихи про весну: Более 100 лучших стихов про весну

Загадка №12

Королева, конь, ладья…
Их в два ряда строю я.
Тут на поле бой пойдет…
Кто игру ту назовет?

(Ответ: Шахматы)

***

Загадка №13

Этот чудо-аппарат
Донесёт быстрее ветра
Голос друга, даже если
Друг — за сотни километров.

(Ответ: Телефон)

***

Загадка №14

Если с верными друзьями
Ты отправился в поход,
Про него скажу я точно:
Он тебя не подведет.
Верный путь домой укажет,
Стрелкой север нам покажет.

(Ответ: Компас)

***

Загадка №15

Я под мышкой посижу
И, что делать, укажу:
Или разрешу гулять,
Или уложу в кровать.

(Ответ: Градусник )

***

Загадка на логику для детей 8 лет

Загадка на логику для детей 8 лет понравятся детям.

Загадка №16

Гусеница не длиннее червяка. Червяк не длиннее гусеницы. Кто длиннее?

(Ответ: Одинаковые)

***

Загадка №17

За чем язык во рту?

(Ответ: За зубами)

***

Загадка №18

У бабушки Ани внук Сережа, кот Пушок, собака Бобик. Сколько у бабушки внуков?

(Ответ: Один)

***

Загадка №19

Сорока летит, а собака на хвосте сидит. Может ли это быть?

(Ответ: сорока летит в небе, а собака сидит на хвосте на земле.)

***

Загадка №20

Что можно приготовить, но нельзя съесть?

(Ответ: Домашнее задание)

***

Загадка №21

Какой прибор человек перед сном включает, а утром выключает?

(Ответ: Будильник)

***

ПОИГРАЕМ В ИГРУ?! Квест: МУЛЬТИМИНИКНИГИЯ


1. Отгадайте Ребус с картинки в этом блоке.

2. Введите ОТГАДАННОЕ слово в поле Код Купона по ССЫЛКЕ и получите книгу бесплатно.

3. Найдите остальные Ребусы в других статьях нашего сайта, чтобы собрать ВСЮ КОЛЛЕКЦИЮ Мини-книг БЕСПЛАТНО!

Загадка №22

У каждого из пяти братьев есть по сестре. Сколько всего у них сестер?

(Ответ: Одна)

***

Загадка №23

Как отыскать черную кошку в комнате, где выключен свет?

(Ответ: Включить свет)

***

Загадка №24

В холодильнике находятся три бутылки сока: виноградный, апельсиновый и томатный. Что нужно будет открыть в первую очередь, если захочется пить?

(Ответ: Дверцу холодильника)

***

Загадка №25

Каким станет спасательный круг зеленого цвета, если бросить его в Красное море?

(Ответ: Мокрым)

***

Загадка №26

Какое число уменьшится на треть, если его перевернуть?

(Ответ: 9)

***

Загадка №27

На дереве висело восемь яблок: три красных и пять зеленых. Через два дня покраснели еще два яблока. Сколько теперь яблок на дереве?

(Ответ: 8)

***

Загадка №28

Что надо сделать, чтобы пять парней остались в одном сапоге?

(Ответ: Чтоб каждый остался в одном сапоге. )

***

Загадка №29

Маленький, серенький на слона похож. Кто это?

(Ответ: Слоненок)

***

Загадка №30

Как правильно говорить: «не вижу белый желток» или «не вижу белого желтка»?

(Ответ: Желток желтого цвета)

***

Смешные загадки для детей 8 лет

Смешные загадки для детей 8 лет тоже очень подходят для этого возраста.

Загадка №31

Как спрыгнуть с десятиметровой лестницы и не ушибиться?

(Ответ: Прыгнуть с первой ступеньки)

***

Загадка №32

Три, три, три и три…Что будет?

(Ответ: Дырка)

***

Загадка №33

Кто получит бесплатный сыр в мышеловке?

(Ответ: Вторая мышка)

***

Загадка №34

Воробей может съесть горсточку зерна, а лошадь не может. Почему?

(Ответ: Воробей слишком маленький, чтобы съесть лошадь)

***

Загадка №35

За что обычно учеников выгоняют из класса?

(Ответ: За дверь)

***

Загадка №36

Во дворе трещит мороз — Шапку ты одень на…

(Ответ: Не на нос, а на голову)

***

Загадка №37

Круглой формы голова,
Той же формы буква…

(Ответ: Не А, а О)

***

Загадка №38

Что нельзя съесть на завтрак?

(Ответ: Обед и ужин)

***

Загадка №39

Холоднющее местечко
В доме нашем это …

(Ответ: Неверный ответ — Печка; Верный ответ — Холодильник)

***

Загадка №40

Кто под проливным дождем не намочит волосы?

(Ответ: Лысый)

***

Загадка №41

Упадет – поскачет, а ударишь – не плачет

(Ответ: Мяч)

***

Загадка №42

Почему корова ложится

(Ответ: Потому что не может сесть)

***

Загадка №43

5 быков дают 10 литров молока. Сколько дадут 10 быков?

(Ответ: Быки не дают молоко)

***

Загадка №44

Как поймать тигра в клетку?

(Ответ: Тигров в клетку не бывает, все тигры в полоску.)

***

Загадка №45

И капризна, и упряма, в детский сад не хочет…

(Ответ: Дочка, не мама.)

***

Загадки для детей 8 лет с подвохом

Загадки для детей 8 лет с подвохом хорошо решаются в этом возрасте.

Загадка №46

Красные двери в пещере моей,
Белые звери сидят у дверей.
И мясо и хлеб – всю добычу мою –
Я с радостью белым зверям отдаю.

(Ответ: Зубы и губы)

***

Загадка №47

У Машиной мамы 5 дочек: Юля, Аня, Оля, Катя. Как зовут пятую дочку?

(Ответ: Маша)

***

Загадка №48

Сколько месяцев в году имеют 28 дней?

(Ответ: Все)

***

Загадка №49

Что случилось 31 февраля?

(Ответ: В феврале 28 (29) дней)

***

Загадка №50

По чему собака бегает?

(Ответ: По земле)

***

Загадка №51

Что у цапли спереди, а у зайца сзади?

(Ответ: Буква «ц»)

***

Загадка №52

Чем оканчиваются день и ночь?

(Ответ: Мягким знаком)

***

Загадка №53

Какой рукой лучше размешивать чай?

(Ответ: В которой ложка)

***

Загадка №54

Сколько будет 2+2*2?

(Ответ: 6)

***

Загадка №55

Что нужно делать, когда видишь зелёного человечка?

(Ответ: Переходить дорогу)

***

Загадка №56

Что в огне не горит и в воде не тонет?

(Ответ: Лед)

***

Загадка №57

Сколько горошин может войти в один стакан?

(Ответ: Ни одной, горошины не могут ходить)

***

Загадка №58

Два брюшка – четыре ушка

(Ответ: Подушка)

***

Загадка №59

Кто грызёт на ветке шишку?
Ну, конечно, это … .

(Ответ: не мишка, а белка)

***

Загадка №60

Я красивый, я летаю,
А весной от солнца таю.
Угадайте поскорей,
Кто же это? … .

(Ответ: не воробей – снег)

***

Интересная загадка для детей 8 лет

Интересные загадки для детей 8 лет предлагают не очевидный ответ.

Загадка №61

Школьный прозвенел звонок,
Завершается урок.
Отдохнуть всех непременно
Приглашает …

(Ответ: Перемена)

***

Загадка №62

На какое дерево садится птица во время сильного ливня?

(Ответ: На мокрое)

***

Загадка №63

Что стоит между окном и дверью?

(Ответ: Буква «И»)

***

Загадка №64

Из какой посуды нельзя ничего поесть?

(Ответ: Пустой)

***

Загадка №65

На ноге стоит одной,
Крутит, вертит головой.
Нам показывает страны,
Горы, реки, океаны.

(Ответ: Глобус)

***

Загадка №66

Белый камушек растаял,
На доске следы оставил.

(Ответ: Мел)

***

Загадка №67

Порой идут, порой стоят,
Порой лежат, порой висят.
А вот сидеть, скажу вам сразу,
Не приходилось им ни разу.

(Ответ: Часы)

***

Загадка №68

Сколько за ней ни иди —
Будет бежать впереди.

(Ответ: Тень)

***

Загадка №69

Любят с искренностью всей
В дом пускать они гостей.
Но в гостях, скажу вам сразу,
Сами не были ни разу.

(Ответ: Двери)

***

Загадка №70

Есть семь братьев:
Годами равные,
Именами разные.

(Ответ: Дни недели)

***

Загадка №71

Хоть у нас четыре ножки —
Мы не мышки и не кошки.
Хоть мы все имеем спинки —
Мы не овцы и не свинки.
Мы не кони, хоть на нас
Вы садитесь много раз.

(Ответ: Стулья)

***

Загадка №72

Без крыльев, а летит,
Без языка, а говорит.

(Ответ: Письмо)

***

Загадка №73

В снежном поле по дороге
Мчится конь мой одноногий
И на много-много лет
Оставляет чёрный след.

(Ответ: Карандаш)

***

Загадка №74

Открыть свои тайны любому готова.
Но ты от неё не услышишь и слова.

(Ответ: Книга)

***

Загадка №75

В поле сестрички стоят,
Жёлтые глазки на солнце глядят.
У каждой сестрички белые реснички.

(Ответ: Ромашки)

***

Загадки про животных для детей 8 лет

Загадки про животных для детей 8 лет кажутся им довольно простыми, но разгадываются также с интересом.


Загадка №76

Она собою хороша.
Её шубеечка рыжа,
Пушистый хвостик
Тоже рыж
И симпатичен.
Только лишь
Живёт рыжуха
Не в норе —
Живёт на дереве,
В дупле.

(Ответ: Белка)

***

Загадка №77

По чему утка плавает?

(Ответ: По воде)

***

Загадка №78

Кто из животных носит каменную рубаху?

(Ответ: Черепаха)

***

Загадка №79

Какая корова, скажите, пока
Ещё никому не дала молока?

(Ответ: Божья коровка)

***

ПОИГРАЕМ В ИГРУ?! Квест: МУЛЬТИМИНИКНИГИЯ


1. Отгадайте Ребус с картинки в этом блоке.

2. Введите ОТГАДАННОЕ слово в поле Код Купона по ССЫЛКЕ и получите книгу бесплатно.

3. Найдите остальные Ребусы в других статьях нашего сайта, чтобы собрать ВСЮ КОЛЛЕКЦИЮ Мини-книг БЕСПЛАТНО!

Загадка №80

Скачет зверушка,
Не рот, а ловушка.
Попадут в ловушку
И комар, и мушка.

(Ответ: Лягушка)

***

Загадка №81

Живут в реке работники,
Не столяры, не плотники,
А выстроят плотину —
Хоть пиши картину.

(Ответ: Бобры)

***

Загадка №82

Ночью ходит, днём он спит.
Если сердится — ворчит.
Он живёт в лесу дремучем.
Сам он круглый и колючий.

(Ответ: Ёж)

***

Загадка №83

Я быстроног, я длинноух,
Хороший мех, чудесный слух.
От псов бегу я без оглядки,
Зато какой храбрец на грядке

(Ответ: Заяц)

***

Загадка №84

Кто, позабыв тревоги,
Спит в своей берлоге?

(Ответ: Медведь)

***

Короткие загадки для детей 8 лет

Загадка №85

Кто на себе свой дом носит?

(Ответ: Улитка или черепаха)

***

Загадка №86

Над рекою я лежу,
Оба берега держу.

(Ответ: Мост)

***

Загадка №87

Что можно увидеть с закрытыми глазами?

(Ответ: Сон)

***

Загадка №88

Каких камней в море нет?

(Ответ: Сухих)

***

Загадка №89

Кто говорит на всех языках?

(Ответ: Эхо)

***

Загадка №90

Не огонь,
А жжется.

(Ответ: Крапива)

***

Загадка №91

Жидкое, а не вода,
Белое, а не снег.

(Ответ: Молоко)

***

Загадка №92

Летит, а не птица,
Воет, а не зверь.

(Ответ: Ветер)

***

Сложные загадки для детей 8 лет

Загадка №93

Какое слово всегда пишется неправильно?

(Ответ: Неправильно)

***

Загадка №94

Задание: У бабушки живут три кошки – кошка Мурка, кошка Клава и кошка Лиза. Обычно они спят на трех разных подушках – желтой, розовой и синей. Кошка Клава любит спать на розовой подушке. Кошка Мурка никогда не выбирает ни розовую, ни синюю. Подумай и скажи, на какой подушке должна спать каждая из кошек?

(Ответ: Клава – на розовой, Мурка – на желтой, Лиза – на синей)

***

Загадка №95

Задание: Вася, Гоша и Витя едят мороженое. Ребята поедают эскимо, фруктовый лёд, и пломбир в вафельном стаканчике. Догадайся, кто какое мороженое ест, если известно, что Вася не любит эскимо, а Витя лакомится фруктовым льдом.

(Ответ: Витя есть фруктовый лед, Вася – пломбир в стаканчике, а Гоша – эскимо)

***

Загадка №96

У кого есть шляпа без головы, нога без сапога?

(Ответ: Гриб)

***

Загадка №97

Что в России на первом месте, а во Франции на втором?

(Ответ: Буква Р)

***

Загадка №98

На березе росло 90 яблок. Подул сильный ветер, и 10 яблок упало. Сколько осталось?

(Ответ: На березе яблоки не растут)

***

Загадка №99

Синенькая шубёнка –
Весь мир покрыла.

(Ответ: Небо)

***

Загадка №100

На что похожа половина апельсина?

(Ответ: На другую половину)

***

Получилось отгадать все загадки для детей 8 лет?

Реально сложные задачи

В отли­чие от преды­ду­щей зада­чи, здесь реше­ние намно­го слож­нее, пото­му что в голо­ве нуж­но дер­жать одно­вре­мен­но 2-3 усло­вия, кото­ры­ми надо про­ве­рять чис­ла. Но мы справимся.

Для реше­ния нам пона­до­бит­ся вспом­нить, что такое про­стые чис­ла и в чём их осо­бен­ность. Про­стое чис­ло — то, кото­рое может делить­ся наце­ло толь­ко на себя и на еди­ни­цу. Напри­мер, чис­ло 5 — про­стое, пото­му что делит­ся толь­ко на 5 и на 1. А чис­ло 6 — не про­стое, пото­му что кро­ме 6 и 1 оно ещё делит­ся на 2 и 3 без остат­ка. Семь тоже будет про­стым чис­лом, а восемь — нет, пото­му что кро­ме 8 и 1 оно делит­ся так­же на 2 и 4.

Если пере­мно­жить два про­стых чис­ла, то полу­чен­ное про­из­ве­де­ние боль­ше никак нель­зя полу­чить дру­гим спо­со­бом (кро­ме умно­же­ния это­го же чис­ла на еди­ни­цу). Пояс­ним на примере.

Возь­мём два про­стых чис­ла 5 и 7 и пере­мно­жим их — полу­чит­ся 35. Боль­ше чис­ло 35 полу­чить никак не полу­чит­ся, кро­ме как умно­жить 35 на 1. Это зна­чит, что если про­из­ве­де­ние мож­но раз­ло­жить на два про­стых мно­жи­те­ля, то дру­гих вари­ан­тов раз­ло­же­ния (кро­ме чис­ла и еди­ни­цы) у него не будет. Это нам при­го­дит­ся при реше­нии задач — и если чис­ло мож­но раз­ло­жить на 2 про­стых, то и их сум­му тоже лег­ко сра­зу посчитать.

Ещё при­мер:

54 = 2 × 27

54 = 3 × 18

54 = 6 × 9, а это зна­чит, что чис­ло 54 нель­зя полу­чить пере­мно­же­ни­ем двух про­стых чисел и нель­зя сра­зу ска­зать, чему одно­знач­но рав­на сум­ма множителей.

И ещё:

21 = 3 × 7

Оба чис­ла про­стые, поэто­му про­из­ве­де­ние 21 мож­но полу­чить толь­ко из них, а зна­чит, лег­ко посчи­тать сум­му — она будет рав­на 3 + 7 = 10.

Теперь пере­ве­дём их диа­лог на язык мате­ма­ти­ки и логи­ки и обо­зна­чим чис­ла как n и m:

Пер­вый: Я понял, что одно из чисел точ­но не про­стое, пото­му что ина­че я сра­зу бы раз­ло­жил чис­ло на про­из­ве­де­ние двух про­стых и лег­ко полу­чил сум­му. А раз так, то это одно из чисел m или n мож­но полу­чить пере­мно­же­ни­ем двух дру­гих чисел. Поэто­му общее про­из­ве­де­ние состо­ит не менее чем из трёх мно­жи­те­лей, при­чём как мини­мум один из них отли­ча­ет­ся от осталь­ных — поэто­му полу­ча­ет­ся несколь­ко вари­ан­тов воз­мож­ных сумм, и я не знаю, какая из них пра­виль­ная (поме­тим это как Пра­ви­ло 1).

Вто­рой: Сум­му, кото­рая у меня есть, нель­зя полу­чить из двух про­стых чисел, поэто­му и твоё про­из­ве­де­ние тоже нель­зя раз­ло­жить на два про­стых мно­жи­те­ля. Это зна­чит, что у меня нечёт­ная сум­ма, пото­му что, по гипо­те­зе Гольд­ба­ха, в нашем слу­чае мож­но полу­чить любое чёт­ное чис­ло, сло­жив два про­стых. А раз это не два про­стых чис­ла, зна­чит, и сум­ма будет нечёт­ная. А ещё эта сум­ма точ­но не рав­на сум­ме двух и про­сто­го чис­ла, пото­му что два — тоже про­стое, ха! Поэто­му есть несколь­ко вари­ан­тов сум­мы m и n, кото­рые под­хо­дят под твои усло­вия, но я не могу пока опре­де­лить, какие имен­но (поме­тим это как Пра­ви­ло 2).

Пер­вый: Из всех мно­жи­те­лей мое­го про­из­ве­де­ния я могу соста­вить толь­ко один вари­ант пары, сум­ма кото­рой подой­дёт под твоё огра­ни­че­ние — не будет раз­би­вать­ся на сум­му двух про­стых или сум­му чисел одно­го мно­жи­те­ля (Пра­ви­ло 3).

Вто­рой: Ах вот как! Из всех вари­ан­тов пар, на кото­рые мож­но раз­бить сум­му и под­хо­дя­щих под твои усло­вия, есть толь­ко одна, кото­рая поз­во­ли­ла бы тебе опре­де­лить это (Пра­ви­ло 4). Теперь и мне понят­но, что это за числа!

Теперь под­бе­рём вари­ан­ты сум­мы, кото­рая была у вто­ро­го. Огра­ни­че­ния такие:

  • нечёт­ная;
  • не рав­на сум­ме двой­ки и про­сто­го числа.

1 — не под­хо­дит, пото­му что оба чис­ла боль­ше единицы.

2, 4, 6, 8… — нет, пото­му что чётные.

3 — нет, пото­му что это сум­ма двой­ки и про­сто­го числа.

5 — нет, по той же при­чине (2 + 3).

7 — тоже нет (2 + 5).

9 — тоже нет (2 + 7, а 7 — про­стое число).

11 — подходит.

13 — нет, пото­му что 13 = 2 + 11 (11 — про­стое число).

15 — нет, пото­му что 15 = 2 + 13 (13 — тоже про­стое число).

17 — подходит.

19 — нет, пото­му что 19 = 2 + 17 (17 — про­стое число).

Спо­соб под­бо­ра сум­мы поня­тен, даль­ше мож­но про­дол­жать по тому же алго­рит­му. Мы же выбе­рем те, кото­рые нам уже подо­шли, и на их при­ме­ре пока­жем, что нуж­но делать даль­ше, что­бы полу­чить пра­виль­ный ответ. Наши чис­ла, кото­рые нам под­хо­дят уже сей­час: 11 и 17. Нач­нём с 11.

Сум­ма = 11.

Най­дём все сла­га­е­мые, кото­рые могут давать эту сумму:

2 + 9

3 + 8

4 + 7

5 + 6

Для каж­до­го из них запи­шем про­из­ве­де­ние и про­ве­рим, выпол­ня­ет­ся ли Пра­ви­ло 3, кото­рое ска­зал пер­вый программист.

Смот­рим на про­из­ве­де­ние 2 × 9 = 18 и как ещё его мож­но получить.

18 = 2 × 9 → Да (Пра­ви­ло 3 выполняется).

18 = 3 × 6 → Нет (Пра­ви­ло 3 не рабо­та­ет, пото­му что 3 + 6 = 9, а 9 мож­но полу­чить из про­стых чисел 2 и 7).

Смот­рим на про­из­ве­де­ние 3 × 8 = 24.

24 = 2 × 12 → Нет (чёт­ная сум­ма, Пра­ви­ло 2 не работает).

24 = 3 × 8 → Да (выпол­ня­ет­ся Пра­ви­ло 3).

24 = 6 × 4 → Нет (чёт­ная сумма).

Смот­рим на про­из­ве­де­ние 4 × 7 = 28.

28 = 2 × 14 → Нет (чёт­ная сумма).

28 = 4 × 7 → Да (выпол­ня­ет­ся Пра­ви­ло 3).

Смот­рим на про­из­ве­де­ние 5 × 6 = 30.

30 = 2 × 15 → Да.

30 = 3 × 10 → Нет (Пра­ви­ло 3 не рабо­та­ет, пото­му что 3 + 10 = 13, а 13 мож­но полу­чить сум­мой про­стых чисел 2 и 11).

30 = 5 × 6 → Да.

Тут мы вооб­ще не можем выбрать одну пару, пото­му что Пра­ви­ло 3 выпол­ня­ет­ся 2 раза, а зна­чит, этот вари­ант отбрасываем.

Полу­ча­ет­ся, что для сум­мы 11 могут быть три вари­ан­та про­из­ве­де­ний, для кото­рых выпол­ня­ет­ся Пра­ви­ло 3: 2 и 9, 3 и 8, 4 и 7. Но тогда Пра­ви­ло 4 не выпол­ня­ет­ся, пото­му что нуж­но, что­бы для одной сум­мы была толь­ко одна пара, кото­рая под­хо­дит под пра­ви­ло 3. Про­дол­жа­ем искать.

Сум­ма = 17.

Най­дём все сла­га­е­мые, кото­рые могут давать эту сумму:

2 + 15

3 + 14

4 + 13

5 + 12

6 + 11

7 + 10

8 + 9

Для каж­до­го из них запи­шем про­из­ве­де­ние и про­ве­рим, выпол­ня­ет­ся ли Пра­ви­ло 3, кото­рое ска­зал пер­вый программист.

Смот­рим на про­из­ве­де­ние 2 × 15 = 30 и как ещё его мож­но получить.

30 = 2 × 15 → Да.

30 = 3 × 10 → Нет (Пра­ви­ло 3 не рабо­та­ет, пото­му что 3 + 10 = 13, а 13 мож­но полу­чить сум­мой про­стых чисел 2 и 11).

30 = 5 × 6 → Да.

Тут мы вооб­ще не можем выбрать одну пару, пото­му что Пра­ви­ло 3 выпол­ня­ет­ся 2 раза, а зна­чит, этот вари­ант отбрасываем.

Смот­рим на про­из­ве­де­ние 3 × 14 = 42 и как ещё его мож­но получить:

42 = 2 × 21 → Да.

42 = 3 × 14 → Да.

42 = 6 × 7 → Нет.

Два раза выпол­ня­ет­ся Пра­ви­ло 3 — отбра­сы­ва­ем пару.

Смот­рим на про­из­ве­де­ние 4 × 13 = 52 и как ещё его мож­но получить.

52 = 2 × 26 → Нет.

52 = 4 × 13 → Да.

Смот­рим на про­из­ве­де­ние 5 × 12 = 60 и как ещё его мож­но получить.

60 = 2 × 30 → Нет.

60 = 3 × 20 → Да.

60 = 5 × 12 → Да.

60 = 6 × 10 → Нет.

Два раза выпол­ня­ет­ся Пра­ви­ло 3 — отбра­сы­ва­ем пару.

Смот­рим на про­из­ве­де­ние 6 × 11 = 66 и как ещё его мож­но получить.

66 = 2 × 33 → Да.

66 = 3 × 22 → Нет.

66 = 6 × 11 → Да.

Два раза выпол­ня­ет­ся Пра­ви­ло 3 — отбра­сы­ва­ем пару.

Смот­рим на про­из­ве­де­ние 7 × 10 = 70 и как ещё его мож­но получить.

70 = 2 × 35 → Да.

70 = 5 × 14 → Нет.

70 = 7 × 10 → Да.

Два раза выпол­ня­ет­ся Пра­ви­ло 3 — отбра­сы­ва­ем пару.

Смот­рим на про­из­ве­де­ние 8 × 9 = 72 и как ещё его мож­но получить.

72 = 2 × 36 → Нет.

72 = 3 × 24 → Да.

72 = 4 × 18 → Нет.

72 = 6 × 12 → Нет.

72 = 8 × 9 → Да.

Два раза выпол­ня­ет­ся Пра­ви­ло 3 — отбра­сы­ва­ем пару.

Полу­ча­ет­ся, что для сум­мы 17 может быть толь­ко один вари­ант про­из­ве­де­ния, для кото­ро­го выпол­ня­ет­ся Пра­ви­ло 3: это 4 и 13. А зна­чит, что Пра­ви­ло 4 тоже выпол­ня­ет­ся и мы нашли нуж­ные числа!

Если вы дочи­та­ли досю­да и всё поня­ли — сни­ма­ем шля­пу. Вы не из тех, кого могут испу­гать вычис­ле­ния и логи­че­ский подход!

Загадки для детей 8 лет с ответами

Дом на улице стоит, Детвора к нему спешит. Несут тетрадки, книжки Девчонки и мальчишки.

На носу сидим, На мир глядим, За уши держимся.

Говорит она беззвучно, А понятно и нескучно. Ты беседуй чаще с ней — Станешь вчетверо умней.

Кто на себе свой дом носит?

Красные двери в пещере моей, Белые звери сидят у дверей. И мясо и хлеб – всю добычу мою – Я с радостью белым зверям отдаю.

Без него домой придёшь — Точно в дом не попадёшь.

Себя он раскрывает, Тебя он закрывает.

Есть в комнате портрет, Во всём на вас похожий. Засмейтесь — и в ответ Он засмеется тоже.

Она собою хороша. Её шубеечка рыжа, Пушистый хвостик Тоже рыж И симпатичен. Только лишь Живёт рыжуха Не в норе - Живёт на дереве, В дупле.

Школьный прозвенел звонок, Завершается урок. Отдохнуть всех непременно Приглашает …

Ежедневно в семь утра Я трещу: «Вставать пора!»

Из горячего колодца Через нос водица льётся.

Над рекою я лежу, Оба берега держу.

Живёт он на кухне, Проворный, худой, Он пол подметает Своей бородой.

Вот гора, а у горы Две глубокие норы. В этих норах воздух бродит, То заходит, то выходит.

Два братца не могут расстаться: Утром — в дороге, Ночью — на пороге.

Этот чудо-аппарат Донесёт быстрее ветра Голос друга, даже если Друг — за сотни километров.

Королева, конь, ладья… Их в два ряда строю я. Тут на поле бой пойдет… Кто игру ту назовет?

Едет он на двух колесах, Не буксует на откосах. И бензина в баке нет. Это мой…

Дали братьям тёплый дом, Чтобы жили впятером. Брат большой не согласился И отдельно поселился.

Если с верными друзьями Ты отправился в поход, Про него скажу я точно: Он тебя не подведет. Верный путь домой укажет, Стрелкой север нам покажет.

Зорко смотрит постовой За широкой мостовой. Как посмотрит глазом красным — Остановятся все сразу.

Русская красавица Стоит на поляне В зеленой кофточке, В белом сарафане.

Железные избушки Держатся друг за дружку, Одна с трубой, Тянет всех за собой.

Быстрая течет река Через годы и века.

Бьют Ермилку по затылку, А он не плачет, только ножку прячет.

Когда за грибами ты в рощу идёшь, Её непременно с собой ты берёшь.

Землю пробуравил, Корешок оставил, Сам на свет явился, Шапочкой прикрылся.

Две новые кленовые Подошвы двухметровые: На них поставил две ноги И по большим снегам беги.

Я под мышкой посижу И, что делать, укажу: Или разрешу гулять, Или уложу в кровать.

Самые сложные, трудные загадки на логику с ответами

Загадки могут быть самыми разными: сложными (трудными), легкими, смешными, и не очень, и могут быть использованы для разных целей. Мы привыкли воспринимать их как невинное детское развлечение, которое может скрасить свободное время и немного отвлечь ребенка, на самом деле, подобные задания — это куда более полезная вещь, чем нам кажется, а именно самые сложные загадки которые нуждаются ответов.


Человек при жизни получает это трижды: два раза абсолютно бесплатно, на третий раз ему приходится за это платить. Догадайтесь, о чем идет речь.

О зубахОТВЕТО зубах

В каком случае, смотря на цифру 2, человек произносит «десять»?

Когда на электронных часах 22:00ОТВЕТКогда на электронных часах 22:00

Дороги имеются — ехать нельзя,
Земля есть — пахать нельзя,
Луга есть — косить нельзя,
В реках, морях воды нет.

Географическая картаОТВЕТГеографическая карта

Какое колесо не крутится при правом развороте?

ЗапасноеОТВЕТЗапасное

Все меня топчут, а я — всё лучше.

ТропинкаОТВЕТТропинка

Что имеет голову, но не имеет мозгов?

Сыр, лук, чеснокОТВЕТСыр, лук, чеснок

Кто говорит на всех языках?

ОТВЕТОТВЕТЭхо

По чему можно ходить, бегать, ползать, но практически невозможно ездить?

По лестницеОТВЕТПо лестнице

Она может постареть всего за пару часов. Она приносит людям пользу, убивая при этом себя. Ветер и вода могут спасти её от гибели. Что это такое?

СвечкаОТВЕТСвечка

Семь сестер находятся на даче, где каждая занята каким-то делом. Первая сестра читает книгу, вторая — готовит еду, третья — играет в шахматы, четвертая — разгадывает кроссворд, пятая — занимается стиркой, шестая — ухаживает за растениями. А чем занимается седьмая сестра?

Играет в шахматыОТВЕТИграет в шахматы

На берегу моря был камень. На камне было написано слово из 8 букв. Когда богатые читали это слово, они плакали, бедные радовались, а влюбленные расставались. Что это было за слово?

ВременноОТВЕТВременно

Чем больше из неё берёшь, тем больше она становится.

ОТВЕТОТВЕТЯма

Любят, с искренностью всей, в дом пускать они гостей,
Но в гостях, скажу вам сразу, сами не были ни разу.

ДвериОТВЕТДвери

Бывает лёгкой и тяжёлой, но ничего не весит.

Бывает быстрой и медленной, но не ходит, не бегает, не летает.
Что же это?

МузыкаОТВЕТМузыка

Вот короткая загадка:
У кого за носом пятка?

Обувь, туфли, ботинкиОТВЕТОбувь, туфли, ботинки

Что принадлежит вам, однако другие этим пользуются чаще, чем вы сами?

Ваше имяОТВЕТВаше имя

Завязать можно, а развязать не получится.

РазговорОТВЕТРазговор

Что не является вопросом, но требует ответа?

Звонок в двери или по телефонуОТВЕТЗвонок в двери или по телефону

Кто может поднять и передвинуть и коня, и слона?

ШахматистОТВЕТШахматист

Вот так чудо! Вот так диво!
Как сорвался он с обрыва,
Так уже который год
Все никак не упадет.

ВодопадОТВЕТВодопад

Перед каким простым смертным даже президент снимает шапку?

ПарикмахерОТВЕТПарикмахер

Что можно взять в левую руку, но нельзя в правую?

Локоть правой рукиОТВЕТЛокоть правой руки

Что не может увеличить лупа в треугольнике?

ОТВЕТОТВЕТУглы

Какой человек сможет удержать слона?

ШахматистОТВЕТШахматист

У бога есть, у царя нет,
У Бориса спереди,
А у Глеба позади,
У бабы две,
А у девки ни одной!

Буква (Б)ОТВЕТБуква (Б)

На дне моря лежит сундук. В нем все есть, кроме одного. Чего в нем нет?

ПустотыОТВЕТПустоты

Что никогда не врёт?

ЗеркалоОТВЕТЗеркало

Трудные загадки и их польза

Если вам кажется, что загадки могут быть полезны лишь для детей, то вам стоит взглянуть на этот вопрос с другой стороны. Умные загадки, шарады и иные способы заставить работать мозг логически имеют следующие преимущества для людей любого возраста:

  • Развитие логики — даже смешные или короткие загадки несерьезного характера, построенные на принципе поиска закономерно вытекающий связей могут оказаться хорошим отдыхом для мозга между иной деятельностью или, напротив, стать активизатором подобных умений;
  • Улучшение памяти — особенно в этом могут помочь некоторые трудные задания с подвохом, имеющие отсылки и упоминания некоторых хорошо известных всем фактов, однако не сразу приходящие на ум при первом прочтении загадки. Однако постоянное упражнений в данном направлении поможет развить хорошую память;
  • Нестандартное мышление — как правило, самые сложные загадки кажутся нерешаемыми лишь на первый взгляд, а при более детальном рассмотрении становится ясно, что секрет решения состоял лишь в умении взглянуть на загадку с непривычного угла зрения.

Тест на логику, таким образом, могут стать как способом занять свободное время, так и развить интеллектуальные способности, а для подобного времяпровождения далеко не всегда обязательно подбирать самые сложные загадки, главное, чтобы они были интересными.

Очень важно при подборе трудных загадок для отдельно взятого человека или целой группы принимать во внимание несколько факторов:

  • Возраст — в зависимости от этого показателя будут различны и темы подобранных запросов, и уровень сложности. Не стоит давать сложные загадки детям младшего дошкольного возраста, так как они, скорее всего, не смогут с ними справиться, если нет подсказок, а именно вопроси с ответами. Это и отобьет интерес к головоломках и вообще заданиям подобного типа, и может повлиять на самооценку ребенка;
  • Личные интересы и предпочтения — будет разумно включать специально загадки на тему, интересную самому тому, кто разгадывает, чтобы он легко их отгадывал и чувствовал себя вовлеченным в процесс, а также загадки на темы, которые обычно остаются без должного внимания, чтобы привлечь к ним интерес, а не зацикливаться на узкой теме. Это может расширить кругозор и область интересов;
  • Усидчивость и сосредоточенность, то есть умение ребят сидеть на одном месте или необходимость постоянно совершать физические действия. Во втором случае можно комбинировать подвижные игры с головоломками.

При подборе вопросов очень важно обращать внимание, чтобы загадки были с ответами, иначе вы сами рискуете оказаться в неловкой ситуации.

Интересные факты

Для того, чтобы привлечь внимание к интеллектуальным упражнениям, можно рассказать перед началом участникам несколько интересных фактов о том, что они станут умные, что особенно тяжелые загадки, отгаданные ребятами, дадут им шанс получить особенный приз и т.д. Мы же предлагаем такие интересные сведения:

  • Самая древняя и известная в мире головоломка — это паззлы, появившиеся столетиями раньше нашей эры. А история возникновения довольно проста и в то же время занимательна: однажды учитель географии, имя которого не сохранилось в истории, разрезал карту, чтобы предложить ученикам собрать ее по памяти. Так и появилось это развлечение для приятного досуга, состоящее в сборе единой картины из множества деталей;
  • Сейчас мы можем воспринимать шахматы и шашки как способ скрасить досуг или как отдельный вид спорта. Однако изначально эти игры использовались на Востоке при подготовке к битве. Партия в шахматы как сложные загадки решалась по несколько часов двумя игроками, так они развивали логику и способность принимать нестандартные и трудные решения, чтобы в бою обмануть соперника;
  • Мы давали совет выше, подбирать сложные и смешные с ответами, чтобы избежать неловких ситуаций. Но что делать, если у загадки нет разгадки уже много-много лет? Например, хранящаяся в здании ЦРУ скульптура «Криптос» до сих пор не имеет ответа на зашифрованные послания, хотя этим вопросом занимаются на протяжении долгих лет ученые с разных концов света;
  • Кроме развития логики и способности быстро думать, загадки и подобные упражнения могут посодействовать и с некоторыми другими вещами. Например, если у вас есть проблемы с запоминанием названий улиц, имен людей, с ориентированием в родном городе и вообще на местности, то постоянные тренировки разными видами головоломок помогут частично решить это и облегчить вашу жизнь. А регулярные занятие уменьшат риск получить такие сложности в жизни;
  • Особое внимание к логическим задачкам стали проявлять примерно в 9 веке нашей эры в Европе, тогда же вышел первый сборник таких заданий под авторством Алкуина под названием «Задачи для развития молодого ума». Прогресс науки и желание точнее и быстрее познать мир стало развивать интерес и к логическому мышлению, без которого в алхимии и философии делать совсем нечего. Тогда особые умельцы и очень умные люди стали искусственно создавать такие трудные задачки с ответами, а в последствие Алкуин собрал самые лучшие из них (вероятно, самые популярные из дошедших до него) в специальную книгу;
  • 3 тысячи лет назад поэтом Пиндаром из Греции была написана первая словесная головоломка, иначе загадка. Она была составлена в стихотворной форме, в ответе зашифровано некое важное послание, однако до сих пор эту загадку не смог разгадать никто.

Развитие — это то, чем надо заниматься с самого детства, вам необходимо лишь выбрать, какими способами вы хотите заниматься интеллектуальными и логическими упражнениями. Сложные загадки на логику или с подвохом Вам в этом помогут.

 

 

Самые сложные загадки – 24СМИ

Загадки представляют собой выражения, в которых один объект изображается посредством другого. Чтобы понять, что это за объект, человеку следует проявить не только ум, но и смекалку. Одни загадки считаются простыми, они направлены на развитие сообразительности детей дошкольного и школьного возраста. Другие же не под силу даже взрослому человеку. О самых сложных загадках в мире и пойдет речь ниже.

Топ-10 самых сложных загадок на логику и с подвохом для взрослых (с ответами)

10. Паша засунул в бутылку монетку и заткнул бутылку пробкой. Затем он достал монетку, не вынимая пробки и не разбивая бутылки. Догадайтесь, как он это сделал.

Ответ: Он протолкнул пробку внутрь бутылки.

9. Вите и Сереже купили по коробке конфет. В каждой коробке лежит 12 конфет. Витя из своей коробки съел несколько штук, а Сережа из своей съел столько, сколько осталось в коробке у Вити. Отгадайте, сколько конфет осталось на двоих у Вити и Сережи.

Ответ: 12 конфет.
Загадка о конфетах

8. Человек при жизни получает это трижды: два раза абсолютно бесплатно, на третий раз ему приходится за это платить. Догадайтесь, о чем идет речь.

Ответ: О зубах.

7. Дима и Леша играли дома на грязном чердаке без света. Потом они спустились в комнату. У Димы всё лицо было испачкано грязью, а лицо Леши каким-то чудом осталось чистым. Правда, только Леша отправился в ванную комнату умываться. Отгадайте, почему он так поступил.

Ответ: Леша посмотрел на грязное лицо Димы и решил, что и он такой же грязный, поэтому пошел умываться. А Дима ничего не заподозрил, поскольку увидел перед собой чистое лицо Леши.

6. В каком случае, смотря на цифру 2, человек произносит «десять»?

Ответ: Когда на электронных часах 22:00.

5. Человек ехал на своем грузовике. Фары не были включены. Луна не светила. Перед грузовиком женщина в черной одежде переходила дорогу. Догадайтесь, как человек ее увидел.

Ответ: Женщину было отчетливо видно, поскольку это было днем, а не ночью.
Загадка о шляпе и пистолете

4. Человек повесил свою шляпу и, отсчитав 100 метров, отошел на это расстояние с закрытыми глазами. Затем он развернулся и сделал один выстрел в свою шляпу из пистолета, все так же не открывая глаз. И попал. Отгадайте, как ему это удалось.

Ответ: Он повесил свою шляпу на ствол пистолета.

3. Один мальчик любил похвастаться тем, что он задерживает дыхание под водой на 3 минуты. Его друг сказал, что он может провести под водой и 10 минут без специальных приспособлений. Первый мальчик не поверил и предложил ему пари. Второй мальчик согласился и выиграл спор. Объясните, каким образом он победил.

Ответ: Мальчик заполнил стакан водой, поставил его себе на голову и держал на протяжении 10 минут.

2. Позавчера Илье было 17 лет. В следующем году ему будет 20 лет. Догадайтесь, как такое возможно.

Ответ: Если сегодня – 1 января, а День Рождения у Ильи 31 декабря. В этом случае позавчера (то есть 30 декабря) ему было еще 17 лет, вчера (то есть 31 декабря) исполнилось 18 лет, в нынешнем году исполнится 19 лет, а в следующем году — 20 лет.

1. Мужчину находят мертвым в его кабинете. Тело покойного наклонено над рабочим столом, в руке зажат пистолет, на столе лежит диктофон. Полицейские включают этот диктофон и сразу слышат записанное на пленку сообщение: “Я не хочу дальше жить. В этом больше нет никакого смысла…” После этого раздается оглушающий выстрел. Как полицейские моментально догадались, что это убийство, а не самоубийство?

Ответ: Покойный сам не мог перемотать пленку диктофона.
Загадка о собаке перед зеркалом

Если эти загадки не показались вам слишком сложными, попробуйте найти решение для самой сложной загадки без ответа.

Однажды мудрецу задали вопрос:

“Одной собаке дали четкую команду вилять хвостом только в том случае, если она видит другую собаку, которая не виляет хвостом; и наоборот, не вилять хвостом, если она видит собаку, которая виляет хвостом.”

Вопрос заключается в следующем: что она будет делать, чтобы не нарушить команды, если перед ней поставят зеркало?

Интересные загадки для детей

Озадачивать детей и дошкольного и школьного возраста загадками крайне важно и полезно. Отгадывание головоломок ими воспринимается как игра, и в то же время у них развивается нестандартное мышление, расширяются кругозор и восприятие мира.

Сегодня известно огромное количество загадок для детей. Они разбиты по категориям: загадки-шутки с многозначными словами, загадки-шутки с числами, загадки-обманки, ДА-НЕТ-ки, загадки на смекалку для самых сообразительных и др.

Приведем примеры:

1. Отгадайте, как лучше размешивать чай: левой или правой рукой?

Ответ: Лучше размешивать ложкой.

2. Отгадайте, от чего плывет уточка?

Ответ: От берега.
Загадка об уточке

3. Милена обожает животных. У нее 5 кошек, 6 собак, 3 кролика и 2 хомяка. Отгадайте, сколько ног в комнате, когда Милена и ее домашние животные собираются вместе.

Ответ: Всего 2 ноги, ведь у животных лапы, а не ноги.

4. У России это на первом месте, а вот у Германии – на третьем.

Ответ: Буква «р».

А вот загадки, относящиеся к самым трудным для детей:

1. Как человеку не спать 8 дней?

Ответ: Спать ночью.

2. Вы сидите в самолете, впереди видите лошадь, а сзади – автомобиль. Где вы?

Ответ: На карусели.

3. Какой знак нужно поставить между 6 и 7, чтобы результат оказался меньше 7 и больше 6?

Ответ: Запятую.
Загадка про карусель

Не забывайте, что ребенок должен сам захотеть отгадывать более сложные загадки. Это не должно стать для него тяжелым обязательным испытанием. Мотивируйте ребенка, хвалите его, и в этом случае он с удовольствием будет стараться решить все задачки самостоятельно.

Известно, что при приеме ребенка в 1 класс психологи также проводят тесты, используя загадки и головоломки. Поэтому тренируйте память, ум и сообразительность ребенка, направляя его в поисках правильного ответа, чтобы в дальнейшем это не стало для него серьезной проблемой.

Более того, большинство крупных компаний устраивают тесты для соискателей на сообразительность при приеме на работу. Часть теста представляет собой логические загадки, схожие с перечисленными выше. Поэтому понять принцип и научиться их отгадывать полезно в любом возрасте.

Загадки для детского квеста и ребусы: более 25 идей

Вы решили самостоятельно устроить ребенку квест – отлично! С нашими полезными рекомендациями у вас это легко получится: смотрите наглядные загадки для квеста по квартире и на улице для детей, в которых вы зашифруете любые предметы домашнего обихода или уличные объекты.

Оглавление

Для начала посмотрите видео, где мы рассказываем о самых популярных буквенных загадках для квеста:

На нашем Ютуб-канале вы также найдете много других полезных советов для проведения домашних и уличных квестов, переходите и подписывайтесь. Новые полезные видео выходят каждую неделю.

На что обратить внимание при составлении заданий

Ориентируйтесь на возраст детей и их количество. Основные правила, о которых не стоит забывать: чем больше детей, тем проще должны быть головоломки, зато можно сделать их побольше. Так каждый участник получит возможность и разыскать подсказку, и решить задание. То же правило касается и детей 5-7 лет: у них еще мало терпения, чтобы долго разгадывать сложную головоломку, а вот побегать и поискать – самое то. Для ребят 8-13 лет загадки можно сделать более сложными, а ответы – не такими очевидными.

И не забудьте при подготовке квеста составить для себя последовательность тайников по цепочке в том порядке, в котором вы будете прятать подсказки дома или на улице. Каждая подсказка будет указывать на тайник, где спрятана следующая и так далее, а последняя укажет детям, где спрятан подарок или сладости. Подробную пошаговую инструкцию, как самому сделать квест для ребенка, читайте в нашей статье здесь.

Как придумать и сделать головоломки для квеста дома своими руками

Самые популярные загадки для квеста для детей – это ребусы, шифровки и текстовые загадки. Рассмотрим несколько примеров, как они выглядят и как их составить.

Ребусы для квеста

Ребусы про домашние предметы для квеста

Сразу оговоримся, что советуем включать ребусы в квест для ребят, которые уже умеют читать. И перед тем, как загадать ребусы, нужно или убедиться, что ваши дети знают, как они решаются, или описать основные правила решения:

  • ребус может быть комбинированным: нарисован картинкой, написан буквами и цифрами;
  • одна или несколько запятых слева от слова означает, что нужно отбросить с начала слова такое количество букв, сколько запятых стоит. То же самое, если запятые стоят справа от картинки или слова – нужно убирать буквы с конца;
  • если под или над картинкой нарисована перечеркнутая буква – ее нужно отбросить при чтении решения. Если перечеркнутая цифра – такую по счету букву не читаем;
  • так же в слове могут быть переставлены буквы: тогда над словом написан порядок, в котором их нужно читать.

Некоторые ребусы подразумевают только использование букв, предлогов и союзов, означающих взаимодействие между буквами: например, «о» бежит к «а» – читаем «О к А» – Ока. Из многих букв «ы» состоит нарисованный слог «пр». Читаем «ПР из Ы» – призы. В букве «о» внутри вписан слог «да», читаем «в О ДА» – вода. И так далее, используя любые союзы и предлоги.

Больших художественных способностей от вас не потребуется, но будьте внимательны при составлении ребусов и выбирайте не очень сложные картинки (или предусмотрите возможность получить подсказку от вас или другого помощника).

Мы составили для вас ребусы для домашних детских квестов с решением и ответами. Изучите наши примеры, и вы поймете, как составить любые ребусы для поиска подарка в квартире (или воспользуйтесь нашими – нам не жалко).

Ребус про шкаф: рисуем шарф, под ним пишем пояснение: 1 К 2 4. Как решить? Нужно читать первую букву Ш, за ней букву К и вторую и четвертую буквы: А и Ф. Получится «шкаф».

Как составить ребус на сложное слово, например, «холодильник» — та еще загадка. Решение такое: любое длинное слово мысленно всегда можно поделить на два-три маленьких. Ребус про холодильник замечательно получится путем загадывания двух слов: голод (голодный человечек, меняем первую букву на «х») и будильник (отнимаем первые две буквы). Вот как будет выглядеть такая карточка, мы нарисовали здесь все ребусы сразу с пояснением решения и ответом для вашего удобства.

С помощью слов «один» и «ванна» мы решили зашифровать ребус про диван: у «один» отнимаем первую и последнюю букву с помощью запятых. У «ванны» отнимаем последние две буквы.

Ребус про телевизор для квеста выглядит чуточку сложнее, но и интереснее: нарисуем телефон, отнимем (запятыми) у него три последние буквы. А остаток слова – «визор» звучит прямо как «В, сделанная из ОР». Так и нарисуем – букву «в», сделанную из слогов «ор». Выглядит необычно, и нужно подумать, чтобы разгадать!

Самый простой в мире ребус – про стол: ведь это число 100 и буква «л»!

Ребус про зеркало составной и не такой уж простой: первую часть слова зашифруем через «зерно» без двух букв, а «кало» звучит, как будто к букве «а» бежит слог «ло», вот и нарисуем их! Получим ЗЕР+к А ЛО – вышла интересная загадка про зеркало.

Ребус про подушку, наверное, все видели сто раз, от этого он не становится менее клевым: в виде дроби представим, будто под буквой «у» сидит слог «шка». Под У – ШКА. Вот и получилась наша подушка.

Составим картинкой и буквами ребус про духовку: «духи» без последней буквы и буква «о» в слоге «ка». Как видите, мы нарисовали не слишком красиво, но вполне понятно, на скорую руку, чтобы показать вам, что не нужно быть Пикассо для рисования простого ребуса.

А ребус про микроволновку легко получим из «микроба», «волны» и опять же, «о» в «ка», и отнимем в первых картинках ненужные буквы. Такая загадка про микроволновку детям точно понравится!

В решении ребуса про балкон видно, что очень легко он составился из «белки» и буквы «н».

Воспользовавшись рисунком, означающим «кровь», мы загадали ребус про кровать. Обозначили цифрами все буквы слова, а как их читать и какие буквы добавились, поясняет шифр: 1 2 3 4 А Т 5. Взяв буквы слова «кровь», составили «кровать».

Проще всего ребус про стиральную машину для квеста зашифровать сокращенным названием (всё равно по первому слову дети уже догадаются об ответе) «стиралка»: нарисуем кости и палку, у костей отнимем две первые буквы, в у палки заменим первую букву на «р».

Нам помогут рисунки «кот» и «нож», чтобы составить ребус про окно: под картинками мы указали, как переставить порядок букв и какие нужно читать.

Нарисовать ребус про рюкзак несложно: изобразим рюмку и замок и вычеркнем те буквы, которые не нужно читать.

Ребусы про уличные объекты

Чтобы зашифровать ребус про дверь, нарисуем «два», заменим последнюю букву на «е». А в слове «фонарь» вычеркнем первые четыре буквы.

Какие картинки помогут придумать ребус про скамейку для квеста? Нам сразу в голову пришли «скат» и «лейка». Вычеркиваем ненужное, заменяем одну букву – вуаля, скамейка зашифрована.

Три цапли в лесу, или как составить ребус про лестницу для квеста: нарисуем лес, цифру три и цаплю. Заменим одну букву, вычеркнем ненужные буквы, получим «лес+тни+ца».

Загадки для квеста дома (в квартире)

Что, если вам не хочется рисовать ребус или придумывать текстовую загадку? Ничего страшного, есть отличный способ представить подсказку по-другому. Вот наглядные примеры.

Буквы в шариках

Такое задание есть в одном из наших квестов, «Дело о похищенном торте». Вам понадобятся воздушные шарики по количеству букв в слове (или больше, если хотите добавить пустых бумажек – обманок).

Например, загадка про КАМИН, или КОМОД, КОВЕР (да и любое другое слово!) – это просто 5 букв, написанных на листочках, а листочки засунуты в шарики, которые затем надули! У ребят есть два варианта: прочесть букву сквозь шарик или лопнуть его. Если слово длинное – напишите под каждой буквой ее номер в слове. А перед лопанием шаров можно в них поиграть (например, потанцевать с шариками, или не давать им упасть на пол, пока играет музыка, или меняться шарами по воздуху вкруговую между участниками).

Соедини точки

Отличная загадка про несложные в обведении контура предметы, например, загадка для слов «стул», «кресло» или «утюг». Просто нарисуйте и пронумеруйте точки (или обозначьте их буквами алфавита, тогда соединять нужно будет в алфавитном порядке). Причём, алфавит можно использовать, например, английский, или цифры попросить соединить в порядке убывания. Вы сами контролируете сложность задания в зависимости от возраста детей, которые будут проходить квест. Можно распределить задания на более легкие и посложнее, и указывать, кто именно разгадывает каждую данную загадку.

Вот как у нас в новогоднем Мегаквесте была зашифрована вилка: среди всех значков нужно отыскать и соединить только буквы русского алфавита.

Вычеркни повторы

Нарисуйте квадрат и расчертите его на клеточки, для детей помладше – достаточно 6*6 клеточек, для подростков и взрослых можно и 10*10 клеток. Заполните квадрат повторяющимися буквами любого слова по теме вашего квеста или вообще любыми, но не используемыми в слове вашего тайника, его буквы тоже впишите в квадрат. В задании напишите, что нужно вычеркнуть такие-то буквы и увидеть, какие останутся, а затем составить слово.

Например, загадка про шкаф из нашего Диноквеста: вычеркни из таблицы все буквы Д, И, Н, О. Какие буквы остались? Составь из них слово. Вычеркнув все ненужные буквы, дети увидят, что остались буквы слова «шкаф». Так удобно составлять загадки про часы, окно, стол, диван, дверь (то есть короткие названия тайников).

Словесные загадки

Очень интересно придумывать загадки-описания: «Подсказка находится в твоей комнате, на самом большом предмете, на котором лежат два квадратика в мешочках (загадка про подушку)». «Ищи новую подсказку в горячем ящике». Без уточнения придется пораскинуть мозгами, чтобы догадаться, что это загадка про духовку.

Загадки для квеста на улице для детей

Любая уличная загадка должна решаться довольно быстро, лучше сделать их побольше, чем ломать голову по 10 минут над каждым заданием. Все-таки квест на улице требует большей динамики, чем дома. По возможности не используйте мелкие пазлы или разрезные картинки, чтобы части не унесло ветром или они не растерялись. Предусмотрите, чтобы в общественном месте вашу подсказку случайно раньше времени не обнаружили другие дети или прохожие, тут уж лучше спрятать понадежнее. Если погода облачная – защитите подсказки от дождя пакетиком или обклейте их скотчем.

Описание предмета своими словами

Объем текста регулируйте по возрасту детей. Например, для младших загадка про гараж выглядит, как простое описание своими словами: «Ищите подсказку в специальном помещении для машин».

Описание предмета определением из Википедии или книг

Ребенку 8-12 лет лет можно даже загадку про качели составить мудрено: «Подвижное сооружение с осью вращения сооружение для забавы и развлечения, которое устанавливают в парках, дворах и местах отдыха». Придётся подумать, прежде чем бежать на поиски подсказки.

Шифры для квеста дома, в школе или на улице (с ответами и ключами)

У нас в запасе есть разные интересные шифры для детского квеста, покажем вам самые несложные в приготовлении. Такие шифры подойдут и для квеста школьникам: решаются довольно быстро и можно скорее искать новую подсказку – ведь всем хочется поучаствовать, а ждать очереди бывает непросто.

Шифр с картинками

Нарисуйте ключ к шифру: разлинуйте лист на 33 сектора (можно меньше, тогда вы расшифровываете только те буквы, которые используете, плюс еще несколько ненужных). Пусть каждой букве соответствует любой рисунок. В нашем детективном квесте изображения на тему сыщиков и детективов, у вас это могут быть любые рисунки, символы, геометрические фигуры, точки, линии. Главное, чтобы детям был понятен принцип разгадывания – подставить буквы вместо символов.

Шифр из первых букв для квеста

Похож на предыдущий вариант, но здесь изображения не означают конкретную букву, а ребенку нужно взять первую букву от слова, которое означает этот предмет. Нарисовать такой шифр просто: «н» – носки, «е» – ель, «о» – облако, «я» – яблоко» и т.д.

Вот как зашифровано слово «пододеяльник» в нашем новогоднем Мегаквесте. У вас это может быть загадка другого длинного слова, например, «подоконник», «компьютер». Если в слове есть буквы «ы», «ъ» и «ь», напишите их сразу в нужные клеточки.

Шифры для квестов для подростков – более сложные, но и разгадывать их интереснее и под силу старшим ребятам (от 10 лет). Приведем примеры, для вашего удобства шифры с ответами.

Шифр Цезаря

В этой шифровке буквы зашифровываются другими буквами, которые можно найти, отсчитав определенное количество шагов от зашифрованной буквы. Шаг сдвига букв относительно друг друга вы выбираете сами и он одинаков для всех символов.

Как шифровать: напишите на листке алфавит букв, которые будем шифровать (на нашей картинке он красный). Мы решили шифровать сдвигом на 1 букву вправо (вы можете сделать 6 влево и любую другую комбинацию). Тогда вместо А будем писать Я, вместо Б – Я (алфавит для шифрования – синий на нашем фото). Эти два алфавита вы должны оставить на листе – подсказке, это ключ к шифру. А ниже – напишите зашифрованное (синее) слово, ребенок должен перевести его в красные, истинные буквы.

Загадка со словом «телевизор» путём такого сдвига выглядит как СДКДБЗЖНП, а загадка «туалет» – СТЯКДС. Чтобы расшифровать, ребенок будет двигаться на 1 символ вправо, получая вместо С – Т. Вместо Д – Е.

Для подростков и взрослых к этому шифру (не рисуя алфавит) можно указать такой ключ: Б = В, П = Р, Ж = З. Главное, самому внимательно шифровать слово.

Шифр «Решетка»

Чтобы зашифровать слово, напишите его по буквам в клеточки большого квадрата (как на фото), в остальных квадратиках впишите любые другие буквы. Второй такой же по размерам квадрат вырежьте и прорежьте в нем отверстия ровно на тех местах, где находятся нужные для прочтения слова буквы. Разгадывающему нужно будет приложить решетку с дырками к квадрату с буквами именно так, чтобы прочитать слово из 6-8 букв, например, возьмите для загадки слова «рюкзак», «унитаз», «пылесос», «батарея», «кровать», «балкон». Такая загадка потребует смекалки для решения!

Подведем итог

В этой статье мы привели много конкретных примеров с загадками, ребусами и шифрами про предметы домашнего обихода и уличные объекты. Вы можете по нашим рекомендациям придумать свои или взять наши готовые варианты и просто срисовать. Придумать классные, интересные загадки на день рождения ребенку или просто, чтобы повеселиться в выходной – это не сложно! Не забывайте включать в свои квесты (особенно уличные) подвижные игры – детям полезно отвлечься от шифров, выплеснуть энергию и подвигаться. Примеры игр на улице мы рассказываем и показываем в этой статье. Подвижные занятия дома посмотрите в этой статье. 

Играйте с удовольствием, а если хотите всегда иметь в запасе море готовых идей – подписывайтесь на наш нескучный Инстаграм, там мы бесплатно делимся играми, фокусами, поделками на все случаи жизни. Мы желаем вам вдохновения и радостных минут, проведенных с детьми!

30 невероятных загадок для детей, которые знают толк в загадках, автор Kidadl

Загадки – это фантастический способ заставить разум по-настоящему работать.

Что может быть лучше для развлечения детей, чем загадка – сложный вопрос или фраза с двойным или скрытым смыслом, которые нужно решить силой мозга? Сложные головоломки – отличный способ заставить всю семью мыслить нестандартно.

Загадки не только доставляют удовольствие и заставят хихикать всей семьей, но и помогают всем мыслить творчески.Если ваши дети любят сложные загадки, вы обратились по адресу. Мы собрали множество умных загадок и ответов, которые нужно разгадать – так что не торопитесь и посмотрите, сколько из этих самых сложных загадок для детей вы сможете решить – тест мозга, который вы никогда не забудете.

Радость разгадывания загадки не похожа ни на какую другую, поэтому начните проверять своих детей и посмотрите, сколько из этих невозможных загадок они могут разгадать!

Запутанные загадки для проверки вашего мозга

Мы собрали некоторые из лучших загадок, которые когда-либо бросали вызов вашему разуму – прочтите ниже и посчитайте, сколько вы сможете разгадать!

1.Есть две вещи, которые нельзя есть на завтрак. Кто они такие? Ответ: Обед и ужин.

2. Что можно держать правой рукой, но не левой? Ответ: Левый локоть.

3. У женщины пять дочерей. У каждой из дочерей есть брат. Сколько у нее детей? Ответ: Шесть детей. Есть пять дочерей и один брат.

4. Я всегда перед тобой, но ты меня никогда не увидишь.Что я? Ответ: Будущее!

5. Мужчина вышел на прогулку под дождем без куртки, шляпы, капюшона и зонтика. Почему-то ни один волосок на его голове не промок. Как это возможно? Ответ: Он был лысым!

6. Это принадлежит вам, но все остальные пользуются им гораздо чаще. Что это? Ответ: Ваше имя.

7. Я всегда на обеденном столе, но ты никогда не сможешь меня съесть. Что я? Ответ: Тарелки и столовые приборы!

8.На какой вопрос вы никогда не сможете ответить утвердительно? Ответ: Вы еще спите?

9. Что тяжелее, тонна кирпичей или тонна перьев? Ответ: Ни то, ни другое. Они весят одинаково.

10. У отца Джека трое сыновей. Одного зовут Джон, другого – Джош. Как зовут третьего сына? Ответ: Джек.

11. Что размером со слона, но совсем ничего не весит? Ответ: Тень слона.

Отгадывать сложные загадки

Если вы любите сложные загадки, вы попали в нужное место.Готовы принять вызов?

12. Есть слово, которое становится короче, если к нему добавить две буквы. Что это? Ответ: Коротко.

13. У чего есть шея, но нет головы? Ответ : Футболка!

14. Мужчина бреется каждый день, но его борода всегда остается прежней. Как? Ответ: Он парикмахер, бреющий чужие бороды.

15. Есть одноэтажный дом, и все желтое. Стены желтые, двери желтые, кухня желтая, диваны желтые, все желтое.Какого цвета лестница? Ответ: Лестницы нет. Это одноэтажный дом.

16. Как женщина десять дней подряд не спала? Ответ: Потому что она спала ночью, а не днем!

17. Я легок как перышко, но даже самый сильный человек в мире не может удержать меня дольше нескольких минут. Что я? Ответ: Дыхание.

18. Это происходит один раз в минуту, два раза в неделю и один раз в год. Что это? Ответ: Буква «е»!

19.Что ярко-оранжевого цвета, сверху зеленое и звучит как попугай? Ответ: Морковь!

20. Где можно найти города, улицы, магазины и дома, но нет людей? Ответ: На карте.

21. Что можно легко сломать, не коснувшись, не уронив или не подняв? Ответ: Обещание.

22. Что летает без крыльев? Ответ: Время!

Очень сложные загадки

Хотите разгадать самую сложную загадку на свете? У нас есть множество суперсложных загадок и сложнейших головоломок с ответами ниже, которые вы даже можете счесть невозможным.Потратьте немного времени на по-настоящему сложные загадки ниже и посчитайте, сколько вы сможете разгадать!

23. Кто может бегать, но никогда не ходит, имеет рот, но никогда не говорит, имеет голову, но никогда не плачет, имеет кровать, но никогда не спит? Ответ: Река.

24. Два отца и два сына отправились на рыбалку. Каждый из них поймал рыбу. Всего было поймано три рыбы. Как это возможно? Ответ: Было всего три человека. Дед, отец и сын.Это равняется двум отцам и двум сыновьям.

24. Вы можете найти меня на Земле, Марсе, Меркурии и Юпитере, но не на Венере, Плутоне или Нептуне. Что я? Ответ: Письмо Р.

26. Меня достали из шахты и заперли в деревянном ящике. Меня никогда не отпускают, и почти все меня используют. Что я? Ответ: Грифель карандаша.

27. Если 11 + 2 = 1, что тогда 9 + 5 =? Ответ: Эта загадка относится ко времени. 11:00 + 2 часа = 13:00. 9 вечера + 5 часов = 2 часа ночи.

28. Во что действительно легко попасть, но из чего очень сложно выйти? Ответ: Беда.

29. Есть английское слово, которое произносится одинаково, даже если убрать четыре из пяти букв. Что это? Ответ : Очередь.

30. Я не жив, но могу расти. У меня нет легких, но мне нужен воздух, чтобы выжить. У меня нет рта, но вода меня убивает. Что я? Ответ: Огонь!

Загадок, которые озадачивают (с благодарностью)

Эти сложные загадки должны сбить вас с толку.Нет сомнений в том, что они заставят ваш мозг работать на полную, максимальную мощность, если вы надеетесь их решить.

Эти загадки поражают вас деталями со всех сторон и не только бросают вызов вашему мозгу, но и побуждают мыслить нестандартно.

Некоторые из них могут быть сложнее других, и если вы сочли их трудными, мы предоставим ответы. Итак, посмотрите, есть ли у кого-нибудь из ваших знакомых выдающиеся способности решать проблемы, задавая им эти сложные загадки.

Давайте начнем с десяти самых сложных загадок, выбранных Панелью поэта.

Лучшие загадки, чтобы поразить мозг

Загадки для мозга – это как штанга для бицепсов. Они заставляют вас думать, и в результате вы становитесь немного умнее.

1.) Какое слово начинается с E и заканчивается на E, но состоит только из одной буквы?
Конверт.

Не ненавидите, когда ответ кажется очевидным, а только после того, как прочтете его.Ключом к ответу на этот вопрос была интерпретация «одной буквы».

2.) Он короче остальных, но когда ты счастлив, ты поднимаешь его, как лучший. Что это?
Большой палец.

Ответы на многие сложные загадки можно найти в обычных предметах, которые вы используете каждый день – например, в самой короткой пальце на руке – в большом пальце!

3.) Вы можете носить его с собой куда угодно, и он не будет тяжелым. Что это?
Ваше имя.

Есть определенные сложные загадки, в которых одни слова используются больше, чем другие. Некоторые из этих слов и фраз включают ваше имя, вашу тень, свечу, зеркало и ветер.

4.) Чем больше возьмешь, тем больше оставишь. Что я?
Отпечатки пальцев.

Хорошо, это действительно сложно. Я сомневаюсь, что более 99% людей не ответят на этот вопрос правильно, если они не слышали это раньше.

5.) Чем больше, тем меньше видишь.Что я?
Тьма.

Если вы проявите терпение с этим, это очень ответственно. Противоречивая постановка этой сложной загадки очень распространена.

Другие действительно сложные загадки

Иногда ответы на сложные загадки более очевидны, чем вы думаете. Помните об этом, когда будете решать эти задачи.

6.) Что дырявое, но еще много воды держит?
Губка.

Единственные в своем роде существительные, кажется, излюбленные тем, кто задает загадки.Ни один другой легко идентифицируемый объект не обладает свойствами губки. Так что это хорошая загадка.

7.) Слово, которое я знаю, состоит из шести букв, уберите одну букву, и останется двенадцать. Что я?
Десятки.

Загадки с буквами тоже очень популярны. Они могут быть очень сложными. Ключ к ответу на этот вопрос – сосредоточиться на числе 12.

8.) Вы видите лодку, наполненную людьми. Вы смотрите еще раз, но на этот раз вы не видите ни одного человека в лодке.Почему?
Лодка не затонула. Все люди на лодке женаты.

Ага! Слова с несколькими значениями, например «один», помогают составить хорошую загадку.

9.) Бочка с водой весит 60 фунтов. Что нужно положить в него, чтобы он весил 40 фунтов?
Дырка.

Так тяжело, когда вы впервые слышите это, потому что, когда люди думают о «вложении», они думают о чем-то, что добавляют к уравнению. В то время как дыра добавляется, она имеет совершенно другой эффект, как всем известно.

10.) Какое слово из пяти букв останется только после удаления двух букв?
Камень.

Даже если вы начали с предпосылки найти пятибуквенное слово с «одним» в нем, это все равно сложно.

Сложные загадки, чтобы обмануть ваш разум Видео

Сложные загадки с ответами для детей

Эти сложные загадки для детей вызовут много разговоров и азарта.

11.) Что имеет золотые волосы и стоит в углу?
Веник.

12.) Если он у вас есть, и вы показываете это другим людям, меня нет. Что я?
Секрет.

13.) Водитель автобуса идет не в ту сторону на улице с односторонним движением. Он проходит мимо копов, но они его не останавливают. Почему?
Он шел.

14.) Я говорю без уст и слышу без ушей. У меня нет никого, кроме как ожить с ветром. Что я?
Эхо.

15.) Какое слово выглядит одинаково в перевернутом и обратном порядке?
Плавает.

Совет. Если у вас есть немного свободного времени, постарайтесь отгадать голову и отгадать несколько из этих сложных загадок.Они сохранят ваш разум острым.

Мем: сложные загадки

Короткие сложные загадки

Короткие сложные загадки иногда легче запомнить.

16.) Как футболисты сохраняют хладнокровие во время игры?
Стоят рядом с болельщиками.

17.) Какие платья нельзя носить никогда?
Адрес.

18.) Мои кольца не из золота, но я становлюсь все больше, когда старею. Что я?
Дерево.

19.) Что в конце радуги?
Буква W.

20.) Что имеет десять букв и начинается с газа?
Автомобиль.

21.) Не успел сказать, чем сломать. Что это?
Тишина.

Вам могут понравиться Забавные, у меня никогда не было вопросов

Другие примеры загадок

22.) Сколько кирпичей нужно, чтобы построить кирпичное здание?
Один кирпич – «достроить здание».

23.) Что все время проводит на полу, но никогда не пачкается?
Твоя тень.

24.) Откуда вы знаете, что морковь полезна для глаз?
Вы никогда не увидите кроликов в очках.

25.) Что падает, но не ломается? Что ломается, но не падает?
День и ночь.

Вам может понравиться: Сложные математические головоломки

Трудные и трудные загадки для взрослых

Хорошо, наденьте взрослые штаны для этих сложных загадок.

26.) Мужчина выходит под проливной дождь, и ничто не защищает его от дождя. Его волосы не промокают. Как он это делает?
Он лысый.

27.) Майк и Пэт в пустыне. У них обоих рюкзаки. Пэт мертв. Живой Майк открыл свой рюкзак, Пэт закрыл свой рюкзак. Что в пачках?
Парашюты.

28.) Что имеет четыре пальца и большой палец, но не живое?
Перчатка.

29.) Что имеет сердце, которое не бьется?
Артишок.

30.) Ложусь только один раз – когда умру. Что я?
Дерево.

31.) У меня четыре ключа, но нет замков. У меня есть место, но нет места.Вы можете войти, но не можете войти внутрь. Что я?
Клавиатура.

32.) Дай мне еды, и я буду жить. Дай мне воды, и я умру. Что я?
Огонь.

33.) Как разместить десять лошадей в девяти стойлах?
[t] [e] [n] [h] [o] [r] [s] [e] [s] »

34.) Что происходит через минуту, дважды за мгновение, но никогда за тысячу лет?
Письмо М.

Вам может понравиться: Загадки с ответами

Самые сложные загадки

Мы взяли планку от сложных загадок и подняли ее на самую сложную.Удачи; оно вам понадобится.

35.) Ты мой брат, но я не твой брат. Кто я?
Я твоя сестра.

36.) Как можно бросить сырое яйцо с высоты на бетонный пол, не треснув его?
Бетонные полы очень трудно взломать.

37.) Что можно путешествовать по миру, оставаясь в углу?
Штамп.

38.) Что идет вверх по дымоходу вниз, но не по дымоходу вверх?
Зонт.

39.) Есть две сестры.Одна рождает другую, а она, в свою очередь, рождает первую. Кто эти две сестры?
День и ночь.

40.) Что происходит раз в минуту, два раза в мгновение, но никогда за тысячу лет?
Буква «М».

Примечание. Чтобы ускорить процесс, сразу под запросом нет кнопки «показать ответ». Не нужно нажимать, чтобы узнать ответ. Он указан прямо ниже.

Отъезд: что я? Вызов

Хорошие сложные загадки: Настоящие пташки

Отгадывать загадки весело с детьми и семьей.Запишите несколько и испытайте свою группу.

41.) Петух откладывает яйцо на самом верху скошенной крыши. С какой стороны будет скатываться яйцо?
И петухи не откладывают яиц.

42.) Уберите все, а некоторые еще останутся. Что это?
Слово «здоровый».

43.) Вперед я тяжел, но назад нет. Что я?
Вперед тонна, назад нет.

44.) Чем всегда все кончается?
Буква Г.

45.) Где можно закончить книгу, не закончив предложения?
Тюрьма.

46.) Что состоит только из двух слов, а букв из тысячи?
Почтовое отделение.

Ознакомьтесь с дополнительными вопросами.

Обременительные, сложные загадки

47.) Это насекомое, и первая часть его имени – это имя другого насекомого. Что это?
Жук.

48.) От начала вечности до конца времени и пространства, до начала каждого конца и до конца каждого места.Что я?
Буква Е.

49.) Что размером со слона, но совсем ничего не весит?
Тень слона.

50.) Вы можете видеть меня в воде, но я никогда не промокну. Что я?
Отражение.

51.) У чего есть голова, хвост, коричневый, а ног нет?
Пенни.

Отъезд: 125+ Кто больше всего ответит на вопросы

Задорные головоломки

52.) Какая часть лодки больше всего нравится шопоголику?
Парус.

53.) Назовите слово из 7 букв, из которого если вынуть 4 буквы, у вас останется 1?
Кто-то.

54.) Что нельзя сжечь в огне или утопить в воде?
Лед. Он тает вместо того, чтобы гореть в огне, и плавает в воде.

55.) Если вы участвуете в гонке и обгоняете человека вторым, на каком месте вы находитесь?
Второе место.

56.) Джон заходит в бар и просит стакан воды. Затем бармен стреляет в потолок из пистолета.Джон говорит спасибо и уходит. Зачем Джону нужна была вода?
Икает. Бармен использовал метод неожиданности, чтобы избавить его от икоты.

Сложные загадки с простыми ответами

Будьте терпеливы, отвечая на эти трудные вопросы, и ответы могут просто прийти к вам.

57.) Что может заполнить всю комнату, не занимая места?
Свет.

58.) Что существует, когда это есть у одного человека, но перестает существовать, когда это получает другой человек?
Секрет.

59.) Что будет черным, когда вы его получите, красным, когда вы его используете, и белым, когда вы его полностью освоите?
Уголь.

60.) Беннет втолкнул свою машину в отель и потерял состояние. Как?
Он играл в игру «Монополия».

61.) Один рыцарь, ниндзя и пират в лодке. Лодка разбивается. Пират прыгает первым, ниндзя спрыгивает со второго, а кто прыгает с третьего?
Рыцарь. Люди думают, что начало звучит так: «Однажды ночью…», а не «Один рыцарь…»

62.) Сними с меня шкуру – плакать не буду, а ты будешь! Что я?
Луковица.

63.) Если есть три чашки сахара и вы берете одну, сколько у вас?
Один, так как это единственный, который вы забрали, и это все, что у вас есть. Остальные по-прежнему остаются там, где были.

64.) Как съесть слона?
По кусочку за раз.

65.) У меня одна буква, но в моем имени пишется восемь. Что я?
Конверт.

66.) Какие цветы можно найти между носом и подбородком?
Тюльпаны – поняли? «Две губы.”

Сложные головоломки: викторина

Никто не сказал, что это будет легко. Проверьте свою смекалку, разгадав этот сборник сложных загадок.

67.) Что находится в начале вечности и в конце времен, есть в каждой расе, но всегда последнее в очереди?
Буква Е.

68.) Когда я тебе понадоблюсь, ты меня выбросишь. Когда я тебе не нужен, ты возвращаешь меня. Что я?
Якорь.

69.) Ты не хочешь меня, когда у тебя нет меня, но когда у тебя есть я, ты не хочешь потерять меня.Что я?
Иск.

70.) Ходите по живым, они даже не бормочут. Ходят по мертвым, они бормочут и ворчат. Кто они такие?
листьев.

71.) Человек, который изобрел это, не хочет этого для себя. Человек, который его покупает, покупает это не для себя. И человек, которому это нужно, не знает, что ему это нужно. Что это?
Гроб.

Головорезы

72.) Назови мое имя, и меня больше нет. Что я?
Тишина.

73.) Проезжает человек и видит три двери: дверь из бриллианта, дверь из рубина и дверь изумрудного цвета. Что он открывает первым?
Дверь его машины.

74.) Каждый день многие люди со всего мира приезжают ко мне в гости, однако обычно они остаются всего на пару минут. Многие считают меня очень грязным, но мало кто хочет жить без меня. И всякий раз, когда люди приходят ко мне, они показывают часть себя, которую редко показывают другим. Что я?
Туалет.

75.) Сначала выкинул наружу. Затем я приготовил внутри. А потом я съел внешность и выбросил внутреннюю часть. Что я?
Кукуруза.

76.) Какая буква в алфавите самая крутая?
Буква B. Окружена AC.

77.) У меня есть ключи, которые открывают без замков и места, но без места. Вы можете войти, но не можете войти. Что я?
Клавиатура.

78.) Что ты можешь держать в левой руке, но не в правой?
Правый локоть.

Веселые сложные загадки

Когда вы используете свой разум, полезно посмеяться. Эти забавные сложные загадки должны помочь в этом.

79.) Что такое коричневое и липкое?
Палка.

80.) Сколько яблок растет на дереве?
Все яблоки растут на деревьях. (Извини, не ненавидь меня.)

81.) Какое древнее изобретение, которое до сих пор используется в некоторых частях света, позволяет людям видеть сквозь стены?
Окно.

Наклейки сложные

82.) Какое слово в английском языке этому удовлетворяет? Первые две буквы означают мужчину, первые три буквы означают женщину, первые четыре буквы означают великую, а все слово означает великую женщину?
Героиня.

83.) Мужчина ехал из Нью-Йорка в Лос-Анджелес за четыре дня. В конце поездки он обнаружил, что одна из его шин была проколота. Как он смог проехать?
Шина была запасной шиной автомобиля.

84.) Тревор указал на девушку на улице и сказал: «Она дочь единственного ребенка моей бабушки.«Какое отношение имеет девушка к Тревору?
Она сестра Тревора.

85.) Какие фрукты можно использовать в качестве соломинки?
Клубника! (Хорошо, это было не очень сложно.)

Советы по здравому смыслу для решения сложных загадок

Эти предложения могут помочь вам решить больше.

A.) Хорошо подумайте, прежде чем ответить

Эти сложные загадки трудны не просто так. В большинстве случаев первый ответ, который приходит в голову, неверен.Загадки часто придумываются так, что очевидный ответ не является решением. Это называется неверным направлением. Итак, сделайте паузу и рассмотрите альтернативы, прежде чем выпалить свой ответ.

Б.) Практикуйтесь, разгадывая множество сложных загадок

Подобно тому, как повторяющееся использование ваших мышц помогает нам наращивать мышцы, заставляя ваш мозг часто работать над разгадыванием загадок и головоломок, вы улучшаете ваше нестандартное мышление и способность решать проблемы.

C.) Ищите выкройки

Прочитав решения примерно 50 сложных загадок, вы заметите, что появляются некоторые закономерности.Например, есть загадки типа «Буквенное решение» . Вот пример: то, что происходит раз в минуту, два раза в мгновение ока, но не раз в тысячу лет. Ответ: буква М. Или, что в конце радуги? Ответ: Буква W.

.

Существует также подход «Решение загадки» . Например: «Вы водитель автобуса, и у вас 7 пассажиров. Один сошел на Вашингтон. Двое сели на Мэйн-стрит, а трое выехали на Линкольн. Как зовут водителя автобуса? Ответ: Ваше имя (потому что помните: «Ты водитель автобуса).

Загадки включают каламбуры и двойное значение слов в загадку и решение, а иногда и то, и другое. Вот пример: как футболисты сохраняют спокойствие во время игры? Ответ: Стоят рядом с болельщиками.

Понимание этих шаблонов очень поможет вам, поскольку многие сложные загадки похожи по структуре и подходу.

D.) Разбейте вопрос-загадку на части

Иногда помогает разбить сложные загадки, состоящие из нескольких предложений, на части: начало, середину и конец.Не бойтесь просить загадочного человека повторить сложные загадки – даже несколько раз, если необходимо. Внимательное слушание может помочь вам решить эту проблему.

E. Рассмотрите широкий выбор возможных ответов

Не зацикливайтесь на том, что, по вашему мнению, является правильным ответом, пока не изучите множество вариантов.

F. Будьте осторожны с предположениями

Многие сложные загадки играют на стереотипах. Например, врачами являются только мужчины. Ставьте под сомнение все, и вы сможете найти решение!

г.Для сложных загадок помогает увидеть загадку в письме

Ищите улики повсюду. Определенное местоимение может помочь вам найти правильный ответ. Первая строка сложных загадок часто дает сильный ключ к разгадке. Прочтите и перечитайте.

Часто задаваемые вопросы

Вот несколько часто задаваемых вопросов о загадках.

Какое определение для «загадки»?

Вопрос, призванный ввести в заблуждение или сбить с толку. Загадку обычно представляют как задачу, которую нужно отгадать.Пример: что можно путешествовать по миру, оставаясь в углу? Ответ: Печать.

Какая самая старая известная загадка в мире?

Древняя шумерская цивилизация, живущая на территории, ныне известной как Ирак, рассказала эту загадку около 4000 лет назад: «Есть дом. В него входят слепые, а выходят видящие. Что это? Ответ: Школа. (Это говорит о том, какое значение шумеры придавали образованию.)

Какая самая известная загадка?

Загадка сфинкса: «Что происходит на четырех ногах утром, на двух ногах в полдень и на трех ногах вечером?» Ответ: Человек.Четыре ноги относятся к ползанию ребенка, две ноги – к ходьбе среднего возраста, а три ноги – к ходьбе в старости с использованием трости.

Заключение

И готово! Возможно, вы были удивлены, насколько легко могут быть некоторые из этих сложных и сложных загадок. Чем больше вы сталкиваетесь с трудными загадками, тем легче вам отвечать на новые.

Проверьте свою семью и друзей и узнайте, сколько времени у них уйдет на их решение. Приберегите самое сложное для своих ближайших друзей или старшего брата.

Майк О’Халлоран

О’Халлоран, редактор Greeting Card Poet, является соавтором четырех книг по мелочам из серии Smart Attack.

и т.д.

Вы находитесь на странице сложных загадок.

Вам может понравиться:

Лучшие

Хорошо

Приколы

Для детей

Для взрослых

Лучшие шутки о тук-тук-тук

31 сложная загадка – с ответами для взрослых и детей

Испытайте себя, разгадывая по-настоящему сложные загадки с ответами! Думаете, у вас есть все, что нужно, чтобы разгадывать эти сверхсложные загадки? Затем выйдите на арену и приготовьтесь к некоторым из самых сложных загадок.Загадки – отличный способ сохранить остроту ума и проверить навыки аналитического и критического мышления друзей, семьи, детей и студентов. Посмотрите, может ли кто-нибудь из ваших знакомых обладать способностями гениального уровня, решив эти сложные загадки. Загадки – отличное дополнение к вечеринке или хороший ледокол. Они также отлично подходят для того, чтобы просто поддерживать ваш ум активным. В этом сборнике вы найдете 3 раздела: 1) общий набор из сложных загадок, для решения, 2) сложных загадок для детей и 3) сложных загадок для взрослых .Так что растяните этот ум, разогрейте эти ментальные двигатели и приготовьтесь разгадывать очень сложные загадки.

Загадки с ответами

Вы должны держать меня прямо, насколько это возможно, но очень немногие это делают. Большую часть времени я слегка согнут или искривлен. Ваша печаль обычно заставляет меня сгибаться еще больше, но не сгибайте меня слишком долго, иначе я больше никогда не смогу полностью выпрямиться.Что я?

Если 66 = 2, 99 = 2, 888 = 6, 00 = 2, 7777 = 0, 667 = 2, 276 = 1, 833 = 2, то чему равно 2876?

Показать ответ

Три. Каждый кружок в числе эквивалентен единице. Например, числа 9 и 6 эквивалентны единице, потому что в них есть один круг, а число 8 равно 2 из-за двух петель.

Каждый день многие люди со всего мира приезжают ко мне в гости, но обычно они остаются всего на несколько минут. Многие считают меня очень грязным, но мало кто хочет жить без меня. И когда люди приходят ко мне, они открывают мне ту часть себя, которую редко показывают другим. Что или кто я?

Иногда я сияю, иногда я тупой, иногда я большой, а иногда я маленький.Я могу быть острым, могу быть изогнутым и не задавать мне вопросов, потому что, хотя я сообразительный, я недостаточно умен, чтобы ответить вам. Что я?

Считается, что я только одномерный и мельче, чем все может быть, и многие говорят, что я – основа всего, что мы видим. Что я?

Показать ответ

Строка (из теории струн)

Водитель лимузина едет не в ту сторону по улице с односторонним движением.По пути он проходит мимо четырех полицейских, но никто из них его не останавливает. Один даже улыбнулся и помахал рукой. Почему его не остановила полиция?

Показать ответ

Так как водитель лимузина шел пешком, а не за рулем

Как насчет забавных сложных загадок? Любишь посмеяться? Если да, то вам понравятся эти забавные загадки

.

Если все Кригглы – Кригглы, все Боркины – Квумблины, никакие Хогглы – Боркинсы, а все Кригглы – Боркинсы, правда ли, что все Боркины – Кригглы?

Что вы получите, если добавите 2 ежевики и 5 яблок?

Что из следующего является самым большим? Треугольник, круг, квадрат или прямоугольник?

Показать ответ

Прямоугольник, в нем больше всего букв

Я могу быть длинным и коротким, я могу быть черным, белым, коричневым или пурпурным.Вы можете найти меня по всему миру, и я часто являюсь главным событием. Что я?

Как поместить 4 яблока в коробку размером 7х4х2 дюйма?

Показать ответ

Положите 4 Apple iPhone в коробку

Если пятиугольники имеют пять сторон, восьмиугольники имеют 8 сторон, декагоны имеют 10 сторон и шестиугольники имеют 6 сторон.Сколько сторон у обеденного стола в семье из 3 человек?

Показать ответ

Зависит от количества поданных гарниров

Что эквивалентно 4k + 2k + k?

Показать ответ

7000 (k – символ 1000)

Как можно сжечь яблоко, чернику, розу и тыкву, не оставив пепла, но сохранив его запах?

Показать ответ

Горением свечи своим ароматом

Ух ты! Вы прошли весь путь здесь? Ты … совершенно … рок.Возможно, вы просто достойны этих математических загадок

Загадки для детей

Сборник сложных загадок для детей, представленный ниже, больше подходит для детей. Не заблуждайтесь, это все еще непростые загадки, но это не обязательно очень сложные загадки.Может быть трудно удержать внимание ребенка, особенно в школе или в других учебных заведениях. Дети часто считают образование скучным, но загадки – отличный способ сделать процесс обучения увлекательным и увлекательным. И, конечно же, дети любят вызовы. Вас также может заинтересовать наш самый большой выбор загадок для детей. Итак, без лишних слов, вот наша коллекция сложных загадок с ответами для детей.

Вы можете забрать меня, вы можете подбросить меня, вы можете перевернуть меня, вы можете вращать меня, и у людей во всем мире есть разные версии меня.Что я?

Что можно добавить к числу 3, помимо другого числа, чтобы оно стало четным?

Показать ответ

Буква Е слева от него. Соединив его с буквой E на левой стороне, он превратится в восьмерку (E3 = 8)

Я мерцаю, сияю, рассказываю истории и устраиваю шоу. Чтобы заставить меня, вы должны очень быстро перемещать многие произведения искусства от первого до последнего.Что я?

Что есть в золе, древесном угле, тлеющих углях и огне такого, чего просто нет в дыме?

То, что имеет пятно и очень яркое, иногда бывает красным, белым, синим, желтым или зеленым и часто ослепляет.

Если вы бросите бутылку вина, пакет сока и банку газировки кому-нибудь в голову, какая из них повредит меньше всего?

Показать ответ

Банка газировки, потому что это безалкогольный напиток

Я могу грустить, я могу быть счастлив, я могу злиться и плакать, у меня много лиц, а иногда и размеров.Я могу передавать то, что часто невозможно передать словами, и могу перемещаться с места на место со скоростью света. Что я?

Клэр заказала еду для своей семьи из четырех человек в ресторане быстрого питания. Она заказала еду под номерами 1, 3, 7 и 9, но в конечном итоге они вернули всю еду. Это почему?

Нужно больше? У вас большой мозг? Посмотри, сможешь ли ты разгадать эти логические загадки

Загадки для взрослых

Хотя многие сложные загадки из первого раздела также могут уместиться здесь, эти сложные загадки для взрослых с ответами были выбраны не только потому, что они особенно сложны, но и потому, что некоторые из них могут быть более подходящими для зрелого и пожилого ума.А некоторые из этих загадок дети могут просто не понять. Итак, испытайте своих друзей, раздвиньте свои границы и сведите людей с ума этими действительно сложными загадками для взрослых. Вы также можете насладиться нашей самой большой коллекцией загадок для взрослых.

Какая чашка не может вместить воду или другие жидкости и должна быть подходящего размера, иначе в ней может быть много пролива?

Если я неправильно отсчитываю время, это может означать смерть.Я причиняю избиение, которое не причиняет боли, а скорее поддерживает жизнь. Что я ?

Устали от всех этих чрезвычайно сложных загадок с ответами? Расслабься с легкими загадками

Четыре очень полных женщины, одна из Египта, одна из Ливана, одна из Судана и одна британка, очень опечалились, когда потеряли 30 фунтов.Зачем им грустить?

Показать ответ

Потому что в их странах фунты – валюта, а не мера веса

Иногда я стар, а иногда сообразителен, часто я богат и хорош, и мне очень нравится иметь с собой бокал хорошего вина. Что я?

Почему круг и овал имеют только даты, одинаковые по форме?

Показать ответ

Потому что они не прямые

Что общего между яблоками, ежевикой, мандаринами, абрикосами и малиной, помимо того, что они являются фруктами?

Показать ответ

Все названия (или были) названия технической компании или продукта технологической компании

Что номер 1 сказал номеру 11, когда они ходили за джинсами?

Показать ответ

Хороший разрыв, (бедро – простите, ребята, но вы этого не поймете)

Мэри и Джеки уехали в отпуск.У них обоих по 2 отдельных гостиничных номера. В комнатах нет отопления, и Джеки было очень холодно, а Мэри – нет. Не добавляя никакой одежды, не заворачиваясь в какой-либо другой материал и не используя огонь для обогрева, что делала Мэри, чтобы согреться в этой холодной пустой комнате?

Показать ответ

Мэри забилась в угол комнаты, где всегда 90 градусов (вдумайтесь …)

Вау, вы дошли до конца страницы? Ваш мозг должен быть гигантским! Возможно, вы заслуживаете попытки разгадать эти чрезвычайно сложные загадки.

→ Надеемся, из этих коротких сложных загадок и ответов вы получили отличное умственное упражнение! Пожалуйста, поделитесь с теми, кому, по вашему мнению, понравятся эти загадки.

11 самых сложных загадок с ответами. Сможете ли вы их решить?

11 самых сложных загадок с ответами : Как большой любитель загадок, предлагаю в этом посте серию из 13 сложных загадок! Сможете ли вы найти решение этих 13 загадок?

Самые сложные загадки

Бриллианты

Вы должны найти подделку в партии из 9 бриллиантов.Вы знаете, что он легче других. У вас есть весы, и вам разрешено только два взвешивания.

Как найти поддельный бриллиант среди 9?


Шум

Сначала я прихожу тихо, прихожу, не думая об этом, я умираю при рождении, и тот, кто следует за мной, никогда не приходит тихо.


Завтрак

Когда я ем, я расту, а когда я пью, я умираю, кто я?


Десятичное число

Какое трехзначное число даст вам тот же ответ, если вычесть 5 или разделить на 5?


Всегда первым

Всегда первый, никогда не последний,
Ничто без него никогда не закончится,
Потому что ничто не может начаться без него.

Кто он?


День рождения

Мэгги позавчера исполнилось 20 лет. В следующем году ей будет 23. Как такое возможно?


Лампочка и выключатель

В одной комнате 3 лампочки, в другой 3 выключателя. Откуда вы знаете, какие переключатели включают какие лампочки, зная, что вы можете пойти и заглянуть в комнату только один раз.


Язык

Я говорю на всех языках, и моя голова всегда перевернута.

Кто я?


Обратное число

Какое число переворачивается при умножении на 9?


День недели

Это не день после понедельника и не день до четверга. Завтра не воскресенье и вчера не было воскресенья, послезавтра не суббота, а позавчера не среда.
Какой сегодня день?


Океан

Я родился в воде, я живу в воде, но, выйдя из воды, если она снова коснется меня, я исчезну.
Кто я?

Надеюсь, вам понравилось разгадывать эти разные сложные загадки. Не стесняйтесь сообщать мне в комментариях, если вы хотите, чтобы я предлагал подобные статьи!

Если вы хотите разгадывать больше загадок, я могу посмотреть эту книгу, в которой перечислено большое количество загадок ( Как партнер Amazon я получаю прибыль от соответствующих покупок ):

Вернуться в главное меню

30 невероятных загадок, которые очень сложно решить (с ответами) ▷ Туко.co.ke

Вы любите разгадывать головоломки? Человеческая природа – искать решение и понимание сложного вопроса, и невозможные загадки – идеальные головоломки. Может быть, вы хотите потренировать свой ум или просто скоротать время, ломая голову над неразрешимой головоломкой. Хотя интеллектуальные шутки – это весело, их легко решить за секунды. Однако разгадывание загадок занимает немного больше времени, что делает игру еще более увлекательной. При регулярной практике и последовательности загадки могут стать незаменимыми инструментами жизни.Сколько подготовки и оттачивания нужно, чтобы решить сложнейшую головоломку? Возможно, вам придется попытаться разгадать сложные загадки, чтобы разгадать это.

Изображение: pexels.com, (изменено автором)
Источник: UGC

«Разгадай мне это, дай мне загадку…». Такие слова являются синонимами загадок, в основном они используются как вступительные слова. И вам лучше подготовиться к творческому размышлению над загадкой, если вы надеетесь дать соответствующий ответ. Не забывайте, какими бы сложными они ни были, они предназначены для людей.

Читайте также

Лучшие цитаты гангстеров о любви, уважении и жизни

Почти невозможные загадки – рай для тех, кто ищет увлекательные испытания. Такие головоломки просты, но, поскольку они имеют скрытый смысл, многие люди попадают в тупик. Если вы верите в свои методы разгадывания загадок, поздравляю вас и добро пожаловать в путешествие по самым крутым загадкам.

Сборник невозможных загадок – проверьте свои навыки

Вы ищете невозможные загадки с простыми ответами? Конечно, вы весело проводили время, решая самые сложные из них, но хотите получить что-то более сложное и готовое решение, если оно окажется неразрешимым.

Они могут быть вам интересны, чтобы отточить свои навыки решения загадок и стать гуру. Независимо от вашего интереса, вы будете получать удовольствие от идеальных интеллектуальных задач, и неважно, молод вы или стары – эти загадки для тех, кто ищет головорезов.

1. Убийца приговорен к смертной казни. Ему предстоит выбрать одну из трех комнат: первая полна бушующих огней; второй – убийцы с заряженным ружьем; и третьи львы, которые годами не ели.Какая комната самая безопасная?

Читайте также

Популярные цитаты о ненавистниках, ненавидящих ваши отношения

Ответ:

Бедные львы умерли от голода.

2. На рыбалку ходили мальчик и врач. Мальчик был сыном доктора, но доктор не был отцом мальчика. Кто доктор?

Ответ:

Мать мальчика.

3. Что имеет четыре пальца и большой палец, но не живое?

Ответ:

Перчатка.

4. Если у человека пчела в руке, то что у него в глазу

Ответ:

Красота, потому что красота в глазах «пчеловода».

5. Две девушки вместе ужинали. Они оба заказали холодный чай. Одна девушка выпила их очень быстро и закончила за время, которое потребовалось другой, чтобы выпить только одну. Девушка, выпившая одну, умерла, а другая выжила. Все напитки были отравлены. Как выжила девушка, которая больше всего пила?

Ответ:

Яд был во льду, когда она медленно выпила одну рюмку, тающий лед высвободил яд.

Читайте также

Лучшие цитаты Джона Дикаря

ЧИТАЙТЕ ТАКЖЕ: Веселые каламбуры, цитаты и высказывания о пицце

6. Тот, кто меня заставляет, не говорит мне. Кто меня забирает, меня не знает. Кто меня знает, не хочу меня. Что я?

Ответ:

Фальшивые деньги.

7. Как вы называете комнату без окон и дверей?

Ответ:

Гриб.

8. Каждую ночь мне говорят, что делать, и каждое утро я делаю то, что мне говорят.Но все же я не ухожу от твоих ругательств.

Ответ:

Будильник.

9. Я то, что люди любят или ненавидят. Я меняю внешность и мысли людей. Если человек позаботится о себе, я поднимусь еще выше. Некоторых я обманываю. Для других я загадка. Некоторые люди могут захотеть спрятать меня, но я покажу. Как бы люди ни старались, я никогда не проиграю. Что я?

Ответ:

Возраст.

10.Его нельзя увидеть, нельзя почувствовать, нельзя услышать и нельзя понюхать. Он лежит за звездами, под холмами и пустыми ямами, которые заполняет. Он приходит первым и следует за ним, заканчивает жизнь и убивает смех. Что это?

Читайте также

60+ цитат и высказываний о мести

Ответ:

Тьма.

11. Вы измеряете мою жизнь часами, а я служу вам истечением срока. Я быстр, когда худой, и медлителен, когда толстый. Ветер – мой враг.

Ответ:

Свеча.

12. Вы видите лодку, заполненную людьми. Он не затонул, но когда вы посмотрите еще раз, вы не увидите ни одного человека на лодке. Почему?

Ответ:

Все были женаты.

Короткие невозможные загадки

Короткие и сложные загадки непросты, потому что в них не так много текста, чтобы помочь вам найти решение. Они побуждают к логическому мышлению, но вы должны сосредоточиться на получении правильного ответа. Итак, каковы лучшие короткие, неразрешимые загадки и ответы на них?

13.Какое английское слово состоит из трех последовательных двойных букв?

Ответ:

Бухгалтер.

14. Что имеет глаз, но не видит?

Ответ:

Игла.

(У иглы есть «ушко», в котором проходит нить, но она не видит им)

Читайте также

Самые острые любовные цитаты всех времен

ЧИТАЙТЕ ТАКЖЕ: Цитаты Пророка о Рамадане Мухаммад и Коран

15. Что летает без крыльев?

Ответ:

Время.

16. Каков конец радуги?

Ответ:

Буква W.

17. Переверни меня на бок, и я все. Разрежь меня пополам, и я ничто. Что я?

Ответ:

Число 8.

18. Как разместить 10 лошадей в 9 стойлах?

Ответ:

[t] [e] [n] [h] [o] [r] [s] [e] [s]

19. Что можно держать в правой руке, но не держать? оставил?

Ответ:

Правый локоть.

Сложные загадки, которые заставляют вас выглядеть глупо

Изображение: pixabay.com, (изменено автором)
Источник: UGC

Если вы хотите дать своему мозгу тренировку, то решите самые сложные загадки, которые заставят вас выглядеть глупо. В конце концов, вы получите лучшую головоломку для себя и, вероятно, получите решение после нескольких часов работы над своим мозгом.

ЧИТАЙТЕ ТАКЖЕ: Лучшие цитаты и пожелания Рамадана

Читайте также

Лучшие веселые цитаты о жизни в целом

20.Вы выходите из лабиринта, а перед вами три двери. Дверь слева ведет к бушующему аду, а центральная – к смертоносному убийце. Дверь справа ведет к льву, который не ел три месяца. Какую дверь выбрать?

Ответ:

Лев умер бы, если бы не ел в течение трех месяцев.

21. Умер старик, оставив двух сыновей. В своем завещании он приказывает своим сыновьям участвовать в гонках со своими лошадьми, и тот, у кого лошадь будет медленнее, получит его наследство.Два сына участвуют в гонке, но, поскольку они оба сдерживают своих лошадей, они идут к одному мудрецу и спрашивают его, что им делать. После этого братья снова участвуют в гонке – на этот раз полным ходом. Что сказал им мудрец?

Ответ:

После того, как они поменяются лошадьми, любой, кто выиграет скачку, получит наследство, поскольку технически он все еще владеет проигравшей лошадью.

Прочтите также

25 лучших жестоких цитат для девочек

22. Я знаю тысячу лиц и считаю хвостатых голов, ярких на глазах у многих умерших, хорошо владеющих могучей силой, у кого скромный рост .Массы падают на колени, чтобы не видеть мою единственную сторону.

Ответ:

Монета.

23. Есть три сундука, в каждом по 100 монет. В одном сундуке 100 золотых монет, в другом 100 серебряных монет, а в третьем поровну 50 золотых и 50 серебряных монет. Каждый сундук промаркирован, но не все. Вам разрешено выбрать одну монету только из одного из ящиков, и после этого вы должны правильно идентифицировать каждый из трех сундуков. Что вы должны сделать?

Ответ:

Возьмите монету из сундука с надписью 50/50.Если вы получите золотую монету, вы узнаете, что в сундуке есть только золотые монеты. Следовательно, помеченное серебром должно быть сундуком 50/50, а помеченное золотом – серебряным сундуком.

Читайте также

20 самых известных цитат Сократа

24. Вы должны держать меня прямо, насколько это возможно, но очень немногие это делают. Большую часть времени я слегка согнут или искривлен. Ваша печаль обычно заставляет меня сгибаться еще больше, но не сгибайте меня слишком долго, иначе я никогда не смогу снова полностью выпрямиться.Что я?

Ответ:

Ваша осанка.

ЧИТАЙТЕ ТАКЖЕ: Пасхальные послания 2020 для друзей и семьи

Изображение: pixabay.com, (изменено автором)
Источник: UGC

25. Солнце поджигает их, рука ломает их, нога наступает на них, и их вкус во рту. Кто они такие?

Ответ:

Виноград.

26. Когда на меня брызгает жидкость, она не просачивается. Когда меня много двигают, я изрыгаю жидкость. Когда меня бьют, я меняю цвет, и цвета у меня довольно разные.То, что я покрываю, довольно сложно, но я элементарно сгибаюсь. Что я?

Ответ:

Скин.

27. Я двигаюсь очень медленно с незаметной скоростью, хотя я не тороплюсь, я никогда не опаздываю. Я сопровождаю жизнь и переживаю прошлую кончину; Многие женщины смотрят на меня с уважением. Что я?

Читайте также

11 вещей, которые меняют женщины после того, как они говорят, что я делаю, что раздражает мужчин

Ответ:

Я – твои волосы.

28.Врач и водитель автобуса влюблены в одну и ту же женщину, в привлекательную девушку по имени Сара. Водителю автобуса пришлось отправиться в длительную поездку на автобусе, которая продлилась неделю. Перед отъездом он дал Сарре семь яблок. Почему?

Ответ:

Яблоко в день удерживает доктора.

29. Мойщик окон моет окно на 25-м этаже небоскреба, когда внезапно он шлепается и падает. У него нет защитного снаряжения и ничего, что могло бы смягчить его падение, и все же он не пострадал.Как это может быть?

Ответ:

Он был внутри мыть окна.

ЧИТАЙТЕ ТАКЖЕ: Веселые пасхальные цитаты 2020

30. Какую высоту вам нужно будет сосчитать, прежде чем использовать букву А в английском написании целого числа?

Ответ:

Одна тысяча.

Изображение: pexels.com, (изменено автором)
Источник: UGC

Загадки невозможно разгадать, и вы можете бросить своему мозгу идеальный вызов в любой момент, когда почувствуете, что готовы к этому.А кто не любит головоломки? Если вы думаете, что уже являетесь профессионалом в разгадывании сложных словесных головоломок, дайте себе идеальный тест, разгадывая почти невозможные загадки. Они подарят вам идеальный кайф.

Подпишитесь, чтобы смотреть новые видео

ЧИТАЙТЕ ТАКЖЕ:

  • Короткие пасхальные цитаты из Библии
  • 10 мощных молитв о защите от болезней и болезней
  • 100+ лучших статусных цитат в WhatsApp

Источник : Tuko Kenya

Загадки, заставляющие задуматься

Поделиться – это забота!

Загадки – это всегда забавный способ развить мозг.Они отличные ледоколы и создатели логики. Вы знали, что они даже полезны? Вы не ослышались, загадки полезны для здоровья. Вот несколько забавных загадок, которые заставят вас задуматься! Самые важные игры для мозга даже помогут поднять ваш IQ!

Вот почему я решил собрать несколько фантастических загадок, потому что А. Их просто интересно читать, а Б. Загадки полезны для вас!

Так что читайте дальше, чтобы разгадывать фантастические загадки, которые заставят вас задуматься.

У мамы и папы четыре дочери, и у каждой дочери по одному брату.Сколько человек в семье?

Вот несколько отличных загадок, которые помогут вам разобраться. Готовый ? Готово? Посмотрим, на сколько вы сможете ответить!

Загадки, заставляющие задуматься

Всегда перед вами, но вас не видно?

Ответ: Будущее

Какая буква алфавита содержит больше всего воды?
Ответ: C

Облако – моя мать, ветер – мой отец, мой сын – прохладный ручей, а моя дочь – плод земли.Радуга – это моя постель, земля – ​​мое последнее пристанище, а я – человеческое мучение. Что я?

Ответ: Дождь

Меня вытащили из шахты и заперли в деревянном ящике, из которого меня так и не выпустили, но, тем не менее, меня используют почти все. Что я?

Ответ: Грифель карандаша

Было соревнование, где участники должны были что-то провести. По итогам мероприятия победителем стал человек с ограниченными физическими возможностями (у него не было рук и ног)! Что должны были держать конкурсанты?

Ответ: Их дыхание.

Сможете ли вы разобраться в этом?

Вот ответ на эту сложную головоломку

Почему бы человеку, живущему в Нью-Йорке, не похоронить в Чикаго?
Ответ: Потому что он еще жив

Человек, который это делает, в этом не нуждается. Тот, кто покупает это, не использует. Человек, который его использует, не знает, что они есть. Что это?
Ответ: Гроб.

Что нужно сломать, чтобы использовать?
ОТВЕТ: Яйцо

Более сложные загадки

Что начинается с P , заканчивается E и содержит тысячи букв?
Ответ: Почтовое отделение

Что становится влажнее при высыхании?
Полотенце

Что начинается с T , заканчивается на T и содержит T ?
Ответ: Чайник

Ответ на вопрос выше прост: он родился в комнате 1955 года 🙂 Иногда вещи не такие, какими кажутся.

У чего есть лицо и две руки, но нет рук или ног?
Ответ: Часы

Какое пятибуквенное слово становится короче, если к нему добавить две буквы?

Ответ: Короткий

Какое слово начинается и заканчивается на E , но состоит только из одной буквы?
Ответ: Конверт

У чего есть шея, но нет головы?
Ответ: Бутылка

Почему мальчик закопал фонарик?
Ответ: Потому что сели батарейки

Сколько букв в английском алфавите?
Ответ: 18: 3 в , 7 в английском и 8 в алфавите .

В каком месяце 28 дней?
Ответ: Все, конечно

Трое мужчин были в лодке. Он перевернулся, но только у двоих намокли волосы. Почему?

Ответ: Один был лысым

Вот еще один:

Ответ: 7 У каждой дочери есть брат, это означает, что есть только 1 брат + 4 дочери = 5 + мама и папа = 7!

Вот еще одна загадка:

Не можете понять ответ? Иди сюда.

Ищете другую забавную загадку? Проверьте это!

Не можете понять ответ? Зайдите сюда, чтобы увидеть ответ на эту и другие забавные загадки.

Сохранить

Самая сложная загадка – Plentifun

Хотели бы вы прочитать и разгадать самую сложную загадку? Следующая статья раскроет самую сложную загадку в мире, которая заставит работать ваши серые клетки…

Загадки – отличный способ потренировать свои серые клетки и проверить свое присутствие духа. Видно, что у самой сложной загадки есть самый простой ответ, который вы можете себе представить.Эти загадки для мозга с ответами заставят вас хотеть большего и будут проверять не ваш интеллект, а ваш здравый смысл. Следующая статья раскроет самую сложную загадку, которая заставит вас задуматься над ответами.

Самая сложная загадка в мире

Когда дело доходит до решения, какая загадка является самой сложной, становится довольно сложно определить одну-единственную головоломку. Тем не менее, нам в Plentifun удалось найти интересную загадку, которую можно отнести к разряду сложнейших.

Хотите написать для нас? Что ж, мы ищем хороших писателей, которые хотят распространять информацию. Свяжитесь с нами, и мы поговорим …

Давайте работать вместе!

Первая приведенная ниже загадка – самая сложная загадка в мире. Говорят, что разгадывать эту загадку давали выпускникам Гарварда и воспитанникам детских садов. В результате 97% выпускников Гарварда не смогли ответить на вопрос, тогда как 84% учеников детского сада смогли ответить на вопрос.Попробуйте решить следующую самую сложную загадку в мире. Вы даже можете попробовать задать своим детям головоломки и загадки, чтобы немного повеселиться.

? Я превращу белых медведей в белых
И заставлю тебя плакать.
Я заставляю парней писать
А девушки причесываются.
Я выставляю знаменитостей глупыми
А нормальные люди выглядят как знаменитости.
Я превращаю блины в коричневый цвет
И заставляю ваше шампанское пузыриться.
Если сжать меня, я лопну.
Если вы посмотрите на меня, вы лопнетесь.
Сможете ли вы ответить на эту загадку?

(Простейшим альтернативным ответом на загадку выше может быть «НЕТ», поскольку именно это и задает загадка. Поскольку большинство не может ответить на вопрос, ответ остается «Нет». Третьим альтернативным ответом может быть «ВРЕМЯ» на основе стихотворения. с одноименным названием.)
Ответ: Давление

? Если вы посмотрите, вы меня не увидите. И если вы видите меня, вы не можете видеть ничего другого. Я могу сделать все, что угодно, но потом все вернется на круги своя.Что я?
Ответ: Ваше воображение

Самая сложная загадка

Все еще жаждете еще еды для мозга? Следующие ниже действительно сложные загадки когда-нибудь будут дразнить вас и заставят изо всех сил пытаться найти ответ. Просто прочтите эти хитрые загадки для взрослых и узнайте, умнее ли вы детского сада.

? Если курица говорит: «Все куры лжецы», говорит ли курица правду?
Ответ: Цыплята не могут говорить

? Мужчина едет в город и оказывается на развилке дорог.Если он уйдет влево, он отправится в деревню лжецов. Если он пойдет правильно, то он отправится в деревню правды – куда он и хочет попасть. Однако он не знает, какой путь есть.

У него нет времени пройти оба маршрута, поэтому он подходит к незнакомцу, который стоит посреди развилки. Незнакомец говорит, что может задать только 3 вопроса, и он на них ответит.

Хотите написать для нас? Что ж, мы ищем хороших писателей, которые хотят распространять информацию. Свяжитесь с нами, и мы поговорим…

Давайте работать вместе!

Мужчина спрашивает: «Вы из деревни истин?» Незнакомец говорит: «Да!» Однако перед этим человеком все еще стоит дилемма: если незнакомец был из деревни правды, он может говорить только правду, но если он был из деревни лжецов, он бы сказал, что был из деревни правды.

Итак, он спрашивает незнакомца: «Ты говоришь правду?» Незнакомец говорит: «Да!» Но, к сожалению, это оставляет мужчину в том же положении, что и раньше.

Сможете разобраться?
Ответ: Человек спрашивает у незнакомца путь обратно в свою деревню.Если незнакомец был из деревни правды, он отвозит его туда. Если бы он был из деревни лжецов, он все равно отведет его в деревню правды, поскольку он был бы вынужден солгать.

? Отец и сын отправились в поездку на машине. Они попали в автокатастрофу. Отец умер, а сын тяжело ранен. Ему нужна была операция, но в больнице, когда приходит врач, доктор говорит: «Я не могу прооперировать этого мальчика, он мой сын». Кто доктор?
Ответ: Врачом была мать мальчика.

Интересные самые сложные загадки

Если вас не устраивает предыдущая самая сложная загадка в мире, вы можете попробовать решить следующие сложные загадки, чтобы решить их с ответами. Эти загадки не только сложны, но и служат хорошей головоломкой. Не отнимая у вас много времени, давайте перейдем к разгадыванию нескольких самых сложных загадок.

? Два копа вошли в комнату без окон и обнаружили мертвого человека, который, очевидно, повесился к потолку, хотя они не могли понять, как это сделать.Под ним не было стула, с которого он мог бы спрыгнуть, или стола. Просто лужа воды. Как он это сделал?
Ответ: Он стоял на глыбе льда.

? На пляже. Звонит колокол, умирает женщина. Что случилось?
Ответ: Колокол был сигналом о том, что вода слишком глубокая, чтобы плавать. Женщина была глухой и не слышала звонка. Она входит в воду, чтобы искупаться, и тонет.

? Серги и Салли однажды ночью сидели в своей семейной комнате.Пока Серги смотрел телевизор, его жена Салли читала. Внезапно электричество отключилось, и Серги решил лечь спать, но Салли продолжала читать. Не пользуясь искусственным освещением, Салли продолжала читать. Как?
Ответ: Салли была слепой и читала шрифтом Брайля.

? Какое английское слово сохраняет прежнее произношение даже после того, как вы уберете четыре из пяти букв?
Ответ: Очередь

? Какой ряд чисел будет следующим в этой серии?

1
11
21
1211
111221
312211
13112221
Ответ: Первая строка описывает следующую строку, а именно: One, One One, Two Ones, One Two One One и т. Д.Таким образом, ответ 1113213211.

? Это больше Бога и больше зла, чем дьявол. Она есть у бедных, она нужна богатым, и если вы ее съедите, то умрете. Что это?
Ответ: Ничего

? Он ходит на четырех ногах утром, на двух ногах в полдень и на трех ногах вечером. Что это?
Ответ: Мужчина (или женщина) ползает на четвереньках как младенец, ходит на двух ногах как взрослый и использует две ноги и трость в старости.

? Я никогда не был, всегда буду. Никто меня никогда не видел и никогда не увидит. И все же Я – всеобщая уверенность, Жить и дышать на этом земном шаре. Что я?
Ответ: Завтра / будущее

? Был зеленый домик. Внутри зеленого дома был белый дом. Внутри белого дома был красный дом. В красном доме было много младенцев. Что это?
Ответ: Арбуз

? Тот, кто делает, продает.Тот, кто его покупает, никогда не использует. Тот, кто его использует, никогда не знает, что он им пользуется. Что это?
Ответ: Гроб

? Я даю вам группу из трех человек. Один сидит и никогда не встанет. Второй ест столько, сколько ему дают, но всегда голоден. Третий уходит и больше не возвращается.
Ответ: печь, огонь, дым

? Если ты сломаешь меня
Я не перестану работать,
Если ты прикоснешься ко мне
Я могу попасть в ловушку,
Если ты потеряешь меня
Ничего не будет иметь значения.
Ответ: Сердце

? Что человек любит больше жизни
Страх больше, чем смерть или смертельная борьба
Что имеют бедные, чего требуют богатые,
и чего желают довольные люди,
Что скупец тратит, а расточительство спасает
И все люди несут в могилу ?
Ответ: Ничего

? Кто бы это ни сделал, не скажет.
Кто его возьмет, того не знает.
Кто это знает, этого не хочет.
Ответ: Фальшивые деньги

? К заключенному пришел посетитель. После этого охранник спрашивает сокамерника, кто был посетитель. Заключенный отвечает: «Братьев и сестер у меня нет, но отец этого человека – сын моего отца». Каковы отношения между заключенным и посетителем?
Ответ: Это означало, что сын посетил отца.

Надеюсь, вы попробовали и ответили на большую часть вышеупомянутой самой сложной загадки. Загадки – самый интересный способ провести время с друзьями и семьей.Вы можете много общаться с людьми, которые пытаются разгадать ответ. Вы можете попробовать загадать сложные загадки и посмотреть, как ваши друзья щекочут свои мозговые клетки. Я надеюсь, что эта статья о самой сложной загадке мира дала вам одни из самых сложных загадок.

.

Добавить комментарий

Ваш адрес email не будет опубликован. Обязательные поля помечены *